8 - Pulmonology

Editors: Schrier, Robert W.

Title: Internal Medicine Casebook, The: Real Patients, Real Answers, 3rd Edition

Copyright 2007 Lippincott Williams & Wilkins

> Table of Contents > Chapter 8 - Pulmonology

function show_scrollbar() {}

Chapter 8

Pulmonology

Marvin I. Schwarz

Acute Respiratory Distress Syndrome

  • What is the definition of acute respiratory distress syndrome (ARDS)?

  • What are the principles of management of the patient with ARDS?

Discussion

  • What is the definition of ARDS?

    Many acute medical conditions such as congestive heart failure, pneumonia, or the acute noninfectious interstitial pneumonias can mimic the clinical

    P.338


    picture of ARDS. A useful definition includes the following: relatively acute appearance of diffuse pulmonary infiltrates, profound hypoxemia (usually requiring mechanical ventilation), pulmonary compliance less than 20 mL/cm H2O (stiff lungs), and a pulmonary capillary wedge pressure less than 18 mm Hg (noncardiogenic edema). Causes include bacterial, viral, and protozoan pneumonias, all forms of shock, aspiration, high-altitude and neurogenic pulmonary edema, transfusion-related acute lung injury (TRALI) which is often postsurgical, trauma, and burns. The underlying histologic appearance is diffuse alveolar damage. This consists of two phases: the early exudative phase demonstrating alveolar edema and intraalveolar fibrin collective known as hyaline membranes and the fibroproliferative phase consisting of fibroblastic proliferative and collagen deposition.

  • What are the principles of management in the patient with ARDS?

    There are few specific medical therapies for ARDS other than the treatment of the underlying cause. Trials of biologic modifiers have been generally disappointing, except in severe sepsis in which human recombinant activated protein C (activated drotrecogin ) reduces mortality rates. Management is mainly supportive and treating the underlying initiating cause in the hope that the lung can return to normal. The specific goals of therapy are to maintain tissue oxygenation (maximize oxygen delivery) while preventing complications resulting from mechanical ventilation, such as barotrauma (pneumothorax), and lung injury stemming from high airway pressures or oxygen toxicity. A recent National Institutes of Health (NIH)-supported study of mechanically ventilated ARDS subjects indicated a lower mortality for those ventilated with tidal volumes of 6 mL/kg versus the standard 12 mL/kg. There is evidence to support that higher respiratory pressures aggravate the lung injury.

Case

A 27-year-old white male motorcyclist is transported to the emergency room after he was involved in a high-speed, head-on collision with an oncoming automobile. At the accident scene, he was poorly responsive; his initial blood pressure was 70/40 mm Hg and injuries included a flail chest on the right, several pelvic fractures, an open fracture of the femur, and a closed, displaced fracture of the left tibia. A central and two peripheral catheters are inserted, and normal saline is administered at maximal rates. His blood is typed and crossmatched. He is also intubated in the emergency room, and a chest tube inserted on the right yields a bloody return. Suction is applied and no air leak is noted. Several units of blood are administered, and abdominal lavage fluid proves bloody. He is rushed to the operating room to undergo a laparotomy, and a liver laceration is found and repaired. During surgery he receives 8 units of whole blood, 5 units of platelets, and 8 units of fresh frozen plasma. His orthopaedic injuries are appropriately treated and he is transferred to the surgical intensive care unit in critical but stable condition. Chest radiographic study confirms that the endotracheal and chest tubes are in good position and reveals a right lower lobe infiltrate thought to be secondary to a pulmonary contusion. The ventilator is initially set at an inspired oxygen concentration (FIO2) of 40%, respiratory

P.339


rate of 12 per minute, and tidal volume of 90 mL in an assist-control mode. Arterial blood gas measurement reveals a pH of 7.46, a partial pressure of carbon dioxide (PCO2) of 3 mm Hg, and a partial pressure of oxygen (PO2) of 59 mm Hg, with an oxygen saturation of 92%. On the second hospital day, 12 hours after admission, he becomes agitated while on the ventilator, his respiratory rate rises to 25 per minute, his minute ventilation increases from 8.5 to 18 L per minute, and airway pressure rises from 20 to 60 cm H2O. A repeat chest radiograph now shows diffuse airspace pattern. Repeat arterial blood gas analysis reveals a PO2 of 39 mm Hg.

  • What is the differential diagnosis of this patient's clinical deterioration?

  • What are the risk factors for ARDS in this patient?

  • How would you manage this patient's hypoxemia?

  • What are the potential problems associated with positive end-expiratory pressure (PEEP)?

  • What is the mortality rate associated with ARDS?

Case Discussion

  • What is the differential diagnosis of this patient's clinical deterioration?

    Several possibilities need to be considered in this setting. First, an infection (pneumonia) must always be ruled out. It is possible that the patient aspirated gastric contents at the accident scene while his level of consciousness was impaired, and this could have injured the lung directly owing to acid aspiration or set the stage for an overwhelming pneumonia. Alternatively, a nosocomial (hospital-acquired) pneumonia could have been acquired in the surgical intensive care unit, although the early onset of his ARDS makes this unlikely. The massive fluid resuscitation could lead to a fluid-overload congestive heart failure syndrome, even despite a normal-functioning heart before the accident. If a cardiac contusion occurred, this would make him more susceptible to this complication. Pulmonary contusions are worth considering, but are usually more localized and develop within several hours. Airway hemorrhage, perhaps stemming from either bronchial fracture or a traumatic intubation, can occur, but is typically associated with bloody secretions when severe. The ARDS could also have resulted from prolonged hypotension or replacement of blood products or TRALI. Regardless, this patient has ARDS and under these circumstances, pathologic study would show diffuse alveolar damage consisting of hyaline membrane formation, alveolar wall edema, and inflammation.

  • What are the risk factors that put this patient at risk for ARDS?

    This patient's risk factors are: hypotension usually prolonged and severe (systolic blood pressure <90 mm Hg); hypertransfusion more than 10 units of blood products in a 24-hour period; aspiration any patient with depressed mental status is at great risk for gastric aspiration (the resulting chemical injury is a common precursor for ARDS). Fat emboli syndrome this syndrome consists of diffuse pulmonary infiltrates, mental status changes, thrombocytopenia, and conjunctival or axillary petechiae. It occurs most often in the presence of severe and multiple long

    P.340


    bone fractures, and is thought to result from fat emboli migrating from the bone marrow to the lungs. However, it occurs usually 24 to 72 hours after admission, making it unlikely in this patient.

    Other risk factors include sepsis, pneumonia, drugs, pancreatitis, lung contusion, toxic fume inhalation, and oxygen toxicity.

  • How would you manage this patient's hypoxemia?

    Acutely, the FIO2 should be increased to 100%. However, the prolonged administration of 100% oxygen for 3 or more days is likely to lead to oxygen toxicity and further worsening of ARDS. Accordingly, the FIO2 should be decreased to the lowest level that achieves an oxygen saturation of 90% to 92%. There appears to be a threshold of 50% to 60%, below which oxygen toxicity is rare. The ventilator shows that tidal volume should be set at 6 mL/kg. If an FIO2 above this range is necessary, then a trial of PEEP is indicated. PEEP appears to improve oxygenation by recruiting collapsed gas-exchange units (atelectasis). Data indicate that placing the patient in a prone position also improves oxygenation, but not necessarily the outcome.

  • What are the potential problems associated with PEEP?

    PEEP may be lifesaving by improving oxygenation and allowing the FIO2 to be lowered to safe levels; however, it is associated with several potential problems. The first is hypotension. High levels of positive intrathoracic pressure impede venous return to the heart and may be transmitted to the pulmonary arteries, causing pulmonary hypertension. Both these factors serve to decrease cardiac output, which precipitates hypotension.

    The risk of barotrauma is greatly increased with PEEP because of the positive airway pressures and may result in pneumothorax or pneumomediastinum. Pneumothorax in a ventilated patient is often a medical emergency because a tension pneumothorax may evolve. PEEP levels above 15 cm H2O are particularly risky. The prophylactic administration of PEEP, before the onset of ARDS, has been shown to be of no value.

  • What is the mortality rate associated with ARDS?

    In 1967, the mortality rate observed for ARDS was 60%. This has decreased to 30% to 40%, mainly due to improved ventilatory management as opposed to the treatment of the ARDS itself. In the survivors, pulmonary function can return to normal, and this usually occurs by 6 months. Persistent abnormalities past this time indicate pulmonary fibrosis and pulmonary impairment.

Suggested Readings

Calfee CS, Matthay MM. Recent advances in mechanical ventilation. Am J Med 2005;118:584 591.

Choc MK. Acute lung injury/adult respiratory distress syndrome: the third Pittsburgh international lung conference. Proc Am Thorac Soc 2005;2:181 245.

Vinant JL, Abraham E. The last 100 years of sepsis. Am J Respir Crit Care Med 2006;173:256 263.

P.341


Asthma

  • What is asthma, and how is it classified?

  • How is asthma diagnosed?

  • What conditions are associated with or may complicate asthma?

Discussion

  • What is asthma, and how is it classified?

    Asthma is not a single entity, but rather a clinical syndrome consisting of (a) an increase in airway resistance to a variety of stimuli; (b) variable airflow obstruction, which is usually reversible, either spontaneously or with treatment; and (c) a chronic, multicellular inflammatory response within the airways that produces patchy bronchial epithelial denudation, submucosal edema, hypersecretion of mucus, and subbasement membrane collagen deposition. The asthmatic response to stimuli may be immediate, occurring within minutes and termed the early asthmatic response, or delayed, arising several hours after exposure and termed the late asthmatic response. The early asthmatic response primarily results from bronchial smooth muscle constriction and the late asthmatic response is characterized by inflammatory cell infiltration and activation. Both patterns may be triggered by exposure to the same stimuli, and may work in concert to produce sustained narrowing of the airway lumen.

    Asthma may be classified on the basis of either the presumptive etiology or symptom severity and the pattern of airflow obstruction. Historically, attempts have been made to classify asthmatic subjects as having either intrinsic or extrinsic disease. Intrinsic asthmatics have no personal or family history of allergies, their immunoglobulin E (IgE) levels are normal, and they have no easily identifiable environmental precipitants of their symptoms. In contrast, extrinsic asthmatics have allergic or atopic histories, their IgE levels are typically elevated, and they have specific antigenic triggers to their asthma. This traditional etiologic classification is now probably obsolete because individual asthmatic subjects commonly exhibit both IgE- and non IgE-mediated responses to broncho provocative stimuli. Therefore, a classification scheme that is instead based on the severity of symptoms and on lung function is more clinically relevant, and provides a framework on which to base a stepwise treatment approach. One proposed classification scheme is presented in Table 8-1.

    Because the severity of an acute asthma attack may be underestimated by both patients and their families, patients are encouraged to use home expiratory flow rate devices, which can more objectively measure asthma severity. Factors that have been associated with an increased risk of asthma mortality include frequent emergency room visits, hospitalization within the previous year, prior life-threatening episodes, a previous need for intubation, a recent

    P.342


    reduction in the corticosteroid dosage or cessation of use, noncompliance with medical therapy, the presence of serious depression or psychosocial behavioral problems, and a lower socioeconomic status.

    Table 8-1 Asthma Classification Scheme

    Asthma Severity Clinical Features Pulmonary Function
    Mild Intermittent, brief symptoms (<1-2 times per week) Expiratory flow rates >80% of predicted
      Rare nocturnal symptoms (<2 times per week) Expiratory flow rate variability <20%
    Moderate Exacerbations (>2 times per week) Expiratory flow rates 60%-80% of predicted
      Nocturnal symptoms (>2 times per week) Expiratory flow rate variability 20%-30%
    Severe Almost daily bronchodilator use Expiratory flow rate <60% of predicted
      Frequent continuous symptoms Expiratory flow rate variability >30%
      Frequent nocturnal awakenings  
      Physical activities limited by symptoms  
      Hospitalization for asthma within the previous year  

  • How is asthma diagnosed?

    Because patients with asthma are a heterogeneous group, the diagnosis requires assessment of a patient's pulmonary function and attention to select details revealed by the medical history, physical examination, and laboratory tests. Historical features important in establishing the diagnosis of asthma include the episodic and variable nature of the airflow obstruction and the reversibility of the obstruction. The most common symptoms cough, wheezing, chest tightness, shortness of breath, and sputum production are nonspecific and by themselves nondiagnostic. The pattern of symptoms may be suggestive, in that nocturnal (and early morning) symptoms are particularly characteristic of asthma. Commonly reported precipitants of bronchospasm include exercise, cold air, environmental allergens, exposure to occupational or chemical irritants, and upper respiratory tract infections. The differential diagnosis of adult wheezing or cough may include mechanical obstruction of the airway (e.g., foreign body, tumor mass, or granulomatous narrowing), vocal cord dysfunction, congestive heart failure, pulmonary embolus, aspiration injury, pulmonary eosinophilia syndromes, and other forms of chronic obstructive pulmonary disease (COPD) (e.g., cystic fibrosis, chronic bronchitis, and emphysema).

    The physical examination findings may be either unremarkable or suggest the presence of air trapping and hyperinflation, with an increased anteroposterior thoracic diameter and a low diaphragm. Wheezing is the most characteristic breath sound of asthma but is an unreliable indicator of severity. Bronchospasm may produce a prolonged expiratory phase with reduced tidal volumes and minimal air movement. In this setting, faint wheezing paradoxically intensifies as airflow improves. Rhonchi and other adventitious

    P.343


    sounds may suggest the presence of secretions in the airways. Signs of severe airflow obstruction may include an increased pulsus paradoxus, supraclavicular retractions with accessory muscle use (sternocleidomastoid and intercostals), and thoracoabdominal paradox (the paradoxical retraction of abdominal musculature with inspiration).

    Pulmonary function testing should be pursued in all patients with suspected asthma. Spirometric findings of reduced expiratory flow rates with a normal inspiratory flow volume curve, lung volumes suggesting increased thoracic gas and residual volumes, and increased airway resistance are all characteristic signs of asthma and may be alleviated by bronchodilator treatment. After an acute exacerbation of asthma, however, pulmonary function may remain abnormal long after the symptoms have returned to their baseline status.

    Additional studies and signs that may be useful in the evaluation of asthma include (a) bronchoprovocation testing with methacholine, histamine, or exercise to document increased airway responsiveness to stimuli; (b) peripheral eosinophilia; (c) increased IgE levels; (d) Charcot-Leyden crystals (crystallized cationic proteins), eosinophils, or Curschmann's spirals (bronchiolar casts of mucus and cellular debris) in the sputum; and (e) a chest radiograph showing hyperinflation or the presence of barotrauma. No single test or battery of tests is appropriate for every suspected case. Selected studies may provide the objective evidence needed to confirm the diagnosis of asthma when the history and physical examination findings are only suggestive.

  • What conditions are associated with or may complicate asthma?

    Several conditions may complicate the asthma syndrome, and they require special consideration.

    Although a person's clinical course is not predictable, unstable asthma develops during pregnancy in approximately one third of asthmatic women, one third experience no change, and symptoms are actually less severe in one third. Poorly controlled asthma during pregnancy may contribute to prenatal mortality, an increased likelihood of prematurity, and low birth weight. Therefore, using medications to obtain optimal control of asthma is appropriate, even if their safety in pregnancy has not been unequivocally proved. An inhaled corticosteroid preparation, selective 2 agonists, appropriately monitored theophylline use, and even systemic corticosteroids can be used when necessary to prevent fetal hypoxia. Medications that should be avoided include -adrenergic compounds, brompheniramine, epinephrine, and some decongestants (oral agonists), antibiotics (tetracycline and ciprofloxacin), and live virus vaccines.

    The likelihood of asthma-related postoperative complications depends on the severity of the patient's airway hyperresponsiveness, the degree of airflow obstruction, and the amount of excess airway secretions at the time of surgery. In addition, endotracheal intubation and the type of procedure performed (thoracic and upper abdominal) may pose an additional risk. Preoperative corticosteroids may be indicated if expiratory flow rates are reduced (<80% of personal best) or if corticosteroids have been required to control asthma in the previous 6 months.

    P.344


    Maintenance of nasal patency may improve lower airway function and asthma control. Although the mechanisms involved in this relationship are not completely understood, nasal obstruction, such as that caused by rhinitis, sinusitis, and nasal polyps, may lead to asthma instability and worsening of symptoms. Nasal 2 agonists and corticosteroids are sometimes useful in treating nasal obstruction.

    Approximately 2% of all cases of asthma are due to occupational exposure to specific sensitizing substances. Proteins, organic compounds, and some inorganic chemicals (metal salts) have been implicated. Once the diagnosis is established, complete avoidance of exposure is mandatory because continued exposure to even minute concentrations may provoke severe and potentially fatal bronchospasm. Also, once well established, occupational asthma may not be completely reversible. The pharmacologic therapy used for this type of asthma is similar to that used for other forms of asthma, but is no substitute for diligent avoidance of exposure to the offending agents.

    Chemical sensitivity may also provoke asthma attacks. Approximately 5% to 20% of adults with asthma sustain severe and potentially fatal exacerbations of asthma after taking aspirin or other nonsteroidal antiinflammatory drugs (triad asthma). Physical examination in these patients may reveal nasal polyps, and symptoms of vasomotor rhinitis may precede the development of aspirin-induced bronchospasm. Less commonly, sulfites, which may be used as a food preservative, and tartrazine, a yellow dye that may be used as a food coloring, have been linked to the occurrence of acute bronchospasm.

    Although gastroesophageal reflux is more common in people with asthma, its relationship to bronchospasm is controversial. Most people with asthma and symptomatic gastroesophageal reflux have hiatal hernias, and the association between the two conditions may be best demonstrated by simultaneously monitoring the esophageal pH and pulmonary function. Medical management consisting of proton pump inhibitors is usually effective in these patients.

Case

A 26-year-old woman presents to the emergency room at 3:00 a.m. complaining of worsening cough with yellow-green sputum, shortness of breath, and wheezing of 5 days' duration. Her symptoms began after an upper respiratory tract infection that was manifested as a low-grade fever, rhinorrhea with postnasal drip, and nasal congestion. She reports poor sleep quality for the last 2 days because of severe coughing and has used over-the-counter nasal sprays and cough suppressants, but without relief. She is 18 weeks pregnant, but has no significant past medical history. Her physical examination reveals that she is diaphoretic and unable to speak in sentences. Her vital signs reveal a respiratory rate of 30 breaths per minute, a heart rate of 120 beats per minute, a temperature of 37 C (98.6 F), and a pulsus paradoxus of 22 mm Hg. Spirometry is attempted but proves poorly reproducible, with a best effort forced expiratory volume in 1 minute (FEV1) of 30% of predicted. The remainder of her examination findings are noteworthy for the presence of supraclavicular retractions with inspiration, diffusely diminished breath sounds with

P.345


scattered, high-pitched inspiratory and expiratory wheezes, and a palpable subcutaneous crepitation over her anterior thorax. She is quite anxious, but alert and cooperative.

  • What additional studies may be important for the proper management of this patient?

  • What are the initial management considerations in this patient?

  • What are the treatment considerations for ongoing management in this patient?

Case Discussion

  • What additional studies may be important for the proper management of this patient?

    The patient's clinical presentation suggests acute, severe bronchospasm, and the immediate focus of the emergency room effort should be therapeutic rather than diagnostic. Although this is the initial episode of asthma for this patient, numerous factors suggest it is a dangerously severe attack. Dyspnea at rest, an inability to speak, and the use of accessory muscles are important observations. Objective measures of attack severity are an increased pulsus paradoxus and expiratory flow rates less than 40% of predicted. The intensity of wheezing is an unreliable indicator. The presence of subcutaneous emphysema suggests an associated pneumothorax or pneumomediastinum. On the basis of this presentation, chest radiography and arterial blood gas measurement are indicated, although treatment should not be delayed to do these. The chest radiographic findings may exclude the diagnosis of pneumonia and delineate the source of the subcutaneous emphysema. A pneumomediastinum can typically be watched without specific therapy, whereas a pneumothorax would likely require insertion of a chest tube with water-seal suction to bring about reexpansion. The arterial blood gas studies would likely show hypoxemia with hypocapnia. Hypoxemia with an elevated alveolar-arterial oxygen gradient is the result of mismatched ventilation and perfusion. Acute bronchospasm results in hyperventilation, and the arterial blood chemistry data should reflect a respiratory alkalosis with a reduced PaCO2. If the attack is severe and prolonged, the PaCO2 may rise as a result of increased dead space ventilation (high ventilation perfusion ratio) and respiratory muscle fatigue. A normal or elevated PaCO2 in the setting of severe airway obstruction suggests impending respiratory failure and warrants intensive care unit observation, with consideration given to mechanical ventilation.

  • What are the initial management considerations in this patient?

    The immediate goals of therapy are to ensure adequate oxygenation and gas exchange while reducing the bronchospasm and the work of breathing. In this case, the patient is breathing for two and fetal hypoxia is an important concern. At a minimum, adequate supplemental oxygen should be given immediately to ensure a PaO2 exceeding 65 mm Hg and an oxygen saturation greater than 90%. The decision to use ventilatory support consisting of intubation and mechanical ventilation is a difficult one, but may be lifesaving in patients with mental status deterioration, worsening respiratory distress from exhaustion, or progressively increasing PaCO2 levels with respiratory acidosis.

    Frequent dosing with an inhaled 2-adrenergic agonist delivered by nebulizer or metered-dose inhaler is the most effective bronchodilator therapy for acute, severe

    P.346


    asthma (status asthmaticus). Asthmatic patients who are initially unresponsive to intensive inhaled therapy may respond to the subcutaneous delivery of 2 agonists, but oral administration is not indicated for acute management. Epinephrine should be avoided in this patient because it is a teratogen.

    In addition to inhaled 2 agonists and supplemental oxygen, systemic corticosteroids should be instituted early in the emergency room management. Corticosteroids reduce airway obstruction by interrupting the inflammatory cascade at one or more critical steps in its genesis, and may also have a synergistic effect on -adrenergic receptor activity. In general, systemic corticosteroids should be considered if significant improvement is not seen within the first 30 to 60 minutes of intensive bronchodilator treatment. Early corticosteroid use has been shown to lead to a reduction in both the rate of hospitalization and the rate of return to the emergency room after discharge. Inhaled corticosteroids are not indicated for the management of acute, severe asthma. Theophylline preparations offer little additional benefit when added to inhaled 2 agonist treatment in the emergency room, but they may augment respiratory muscle function during hospitalization. The use of inhaled 2 agonists, systemic corticosteroids, and even theophylline preparations (with serum levels kept at <12 g/mL) may be considered appropriate in the setting of pregnancy and unstable asthma. Cautious hydration is also appropriate because insensible water losses increase with hyperventilation. The use of antibiotics is commonly reserved for objectively documented infections. The sputum production, although it is yellow-green, does not mandate antibiotic treatment unless there is Gram's stain evidence of a dominant organism.

  • What are the treatment considerations for ongoing management in this patient?

    The optimal management of chronic asthma relies on four interrelated principles: objective assessment of lung function, pharmacologic therapy, environmental control, and patient education. The goals of effective management are to maintain near-normal pulmonary function and physical activity levels, minimize symptoms and prevent exacerbations, and avoid the adverse effects of asthma medications. Spirometry, based on the peak expiratory flow rates or FEV1, provides an objective measure of asthma control and can be useful in adjusting medications (particularly tapering systemic corticosteroids) and assessing the need for intervention. Pharmacologic therapy is typically prescribed in a stepwise manner. In recognition that asthma is a chronic inflammatory disease, trends in therapy have placed a greater emphasis on the use of inhaled corticosteroids or cromolyn as first-line medications, with inhaled 2 agonists used to bring about acute relief of bronchospasm, as needed. Theophylline preparations and oral -adrenergic agonists are often used as second-line agents, and are particularly useful for controlling the nocturnal worsening of asthma. Short bursts of oral corticosteroids are best used in the early treatment of acute, severe exacerbations, and every effort should be made to avoid chronic dependence on oral corticosteroids once the acute attack is controlled.

    In selected cases, the identification and avoidance of specific triggers of bronchospasm may have significant impact on asthma control. Avoidance of aeroallergens (dust mites, cat dander, pollens, and molds), chemicals (sulfites and tartrazine),

    P.347


    certain medications (aspirin, -blockers, and acetylcholinesterase inhibitors), and strong aeroirritants (tobacco smoke, household sprays, and wood smoke) may be helpful for certain patients. Although exercise is a common precipitating factor, the use of inhaled 2 agonists or cromolyn before exercise may minimize the associated bronchospasm. Last, patient education should begin at the time of diagnosis and be encouraged throughout the continued care. Learning to identify important signs and symptoms of asthma, the correct use of the peak expiratory flow rate meter and metered-dose inhaler, and addressing issues related to medication effects and environmental control may minimize patient misunderstandings regarding the ongoing management of asthma. In this patient, a warning regarding the avoidance of -adrenergic agonists until the completion of the pregnancy is also warranted.

Suggested Readings

Busse WW, Lemansker F. Asthma. N Engl J Med 2001;344:350 362.

International report: international consensus report on diagnosis and treatment of asthma. Publication no. 923091. Washington, DC: U.S. Department of Health and Human Services, Public Health Service, National Institutes of Health, June 1992.

McFadden ER. Acute severe asthma. Am J Respir Crit Care Med 2003;168:740 759.

National Asthma Education Program. Executive summary: guidelines for the diagnosis and management of asthma. Publication no. 913042 A. Bethesda, MD: Office of Prevention, Education, and Control, National Heart, Lung and Blood Institute, National Institutes of Health, July 1991.

National Asthma Education Program. Executive summary: management of asthma during pregnancy. Publication no. 933279 A. Bethesda, MD: Office of Prevention, Education, and Control, National Heart, Lung and Blood Institute, National Institutes of Health, October 1992.

Chronic Obstructive Pulmonary Disease

  • What is chronic obstructive pulmonary disease (COPD)?

  • What are the epidemiologic trends in COPD?

  • What is the most commonly held theory explaining the development of emphysema?

  • What are the common signs and symptoms of COPD?

  • What are the common laboratory and radiographic findings in the setting of COPD?

Discussion

  • What is COPD?

    The term COPD is commonly applied to two disorders: emphysema and chronic bronchitis. Most patients with COPD have a combination of these

    P.348


    two conditions. Some authors also include chronic obstructive asthma and other disorders associated with chronic airflow limitation (e.g., bronchiolitis obliterans and bronchiectasis) under the heading of COPD.

  • What are the epidemiologic trends in COPD?

    There has been an approximate 60% increase in the prevalence of COPD since the late 1970s. Although emphysema is a common postmortem finding in adults, its prevalence is strongly correlated with smoking. COPD is more commonly diagnosed in men than women, but as more adolescent girls than boys are beginning to smoke, this trend may change. A heavy smoker exhibits an average decline in FEV1 of 40 to 45 mL per year; this decline is only 20 mL per year in a nonsmoking adult.

  • What is the most commonly held theory explaining the development of emphysema?

    In part, on the basis of observations gleaned in people with 1-antitrypsin deficiency, most authorities believe that the destruction of the alveolar wall and the airspace enlargement seen in the setting of emphysema are due to an imbalance between the proteases and antiproteases in the lower respiratory tree ( 1-antitrypsin being the major protein in this category). Cigarette smoke inactivates the normal antiproteases in people who do not have 1-antitrypsin deficiency.

  • What are the common signs and symptoms of COPD?

    Although the initial complaint is usually dyspnea, some patients seek medical care because of chronic cough or sputum production, wheezing, recurrent pulmonary infections, or, in rare circumstances, weight loss or lower extremity swelling. Early in the disease, physical examination findings may be normal. Later, auscultation of the chest may reveal wheezing, rhonchi, or, in patients with predominant emphysema, decreased breath sounds. Percussion of the chest typically reveals hyperinflation and low diaphragms. In advanced cases, the point of maximal cardiac impulse may be felt in the subxiphoid area. Cyanosis, a right-sided third heart sound (S3), jugulovenous distention, and lower extremity edema are late findings.

  • What are the common laboratory and radiographic findings in the setting of COPD?

    There are no specific laboratory values seen in the setting of COPD. The routine blood count is normal, although COPD patients with chronic hypoxia may show an elevated hematocrit. The finding of eosinophilia should raise the possibility of concomitant asthma. Typically in COPD the flow rates are reduced, the lung volumes are increased due to hyperinflation as measured by increased thoracic gas volume and functional residual capacity, and the diffusing capacity is decreased in emphysema. Reductions in both the FEV1 and forced vital capacity (FVC) are routinely seen, although the FEV1 is reduced out of proportion to the FVC. Early on, the chest radiograph is usually normal. As emphysema develops, the lungs show hyperinflation, flattening of the diaphragms, and an increased retrosternal airspace. Bullae can be seen. The electrocardiogram tends to be normal, except in advanced disease, when it may show low voltage in the limb leads, early R waves in V1 and V2, and peaked P waves (P pulmonale).

P.349


Case

A 65-year-old man is seen because of a 5-day history of progressive shortness of breath and dyspnea on exertion. He also complains of a cough productive of green sputum, as well as vague right-sided chest pain. He has felt feverish at home, but denies any shaking chills, sore throat, nausea, vomiting, diarrhea, edema, or exposure to anyone with a similar illness.

The patient has been smoking two packs of cigarettes per day for the last 30 years. However, he recently decreased his habit to one pack per day. He was seen by a physician approximately half a year ago and was told that he had emphysema. He has not been hospitalized previously. He is a retired bus driver and lives at home with his wife. They have no pets. Although he has noted some dyspnea on exertion over the last 3 to 4 years, he continues to maintain an active lifestyle and can still mow the lawn without much difficulty. He can walk 1 to 2 mi on a flat surface at a modest pace. The patient rarely drinks alcohol. He denies any other significant past medical history, including a history of childhood asthma or allergic diseases, significant cough, sputum production, or exposure to asbestos. His medications include sustained-release theophylline and over-the-counter vitamins.

On physical examination, the patient is found to be a somewhat thin but well developed and in moderate respiratory distress. His blood pressure is 150/98 mm Hg with a pulsus paradoxus of 20 mm Hg, his pulse is 110 beats per minute, his temperature is 37.9 C (100.22 F) orally, and his respiratory rate is 24 breaths per minute and labored. Head, eye, ears, nose, and throat findings are unremarkable. No adenopathy is found in his neck, and the neck veins are flat. His chest is hyperexpanded, and there is use of the accessory muscles of respiration. Hyperresonance to percussion is noted. His breath sounds are distant with an occasional scattered wheeze. During the cardiac evaluation, the point of maximal impulse is located in the epigastric area. There is a regular tachycardia with a systolic fourth sound (S4) heard best at the right lower sternal border. No murmurs or rubs are noted. His abdomen is scaphoid, bowel sounds are normal, and no tenderness or organomegaly is noted. His extremities are free of clubbing, cyanosis, and edema. Pulse oximetry shows a 91% saturation on room air.

  • What tests and studies would you order in this patient?

    A chest radiographic study reveals the presence of hyperexpanded lung fields, a small cardiac silhouette, evidence of bullous disease in both lungs, and an alveolar infiltrate in the right middle lobe with some degree of volume loss. No effusions are seen.

    Arterial blood gas measurement performed on room air reveals a pH of 7.50, a PaCO2 of 23 mm Hg, a PaO2 of 51 mm Hg, and an oxygen saturation of 92%. Respiratory alkalosis is present with hyperventilation. Results of a complete blood count are as follows: white blood cells, 14,300/mm3 with 8% band forms and 8.4% polymorphonuclear leukocytes; and the hematocrit reading is 44%. A chemistry panel reveals the following findings: sodium, 139 mEq/L; potassium, 4.1 mEq/L; chloride, 108 mEq/L; bicarbonate, 20 mEq/L; blood urea nitrogen, 21 mg/dL; and creatinine, 0.9 mg/dL. His theophylline level is 3.7 g/mL. The electrocardiogram reveals sinus tachycardia with low voltage in the limb leads, and no acute changes.

  • What is your diagnosis based on the information you have, and how would you manage this patient?

  • P.350


  • What therapy should you institute while the patient is in the hospital?

    The patient is started on inhaled 2 agonists and intravenous ampicillin. After 2 days of treatment, his condition fails to improve and respiratory fatigue requiring emergent endotracheal intubation and ventilation develops. His wife states that she does not want to prolong the patient's life by artificial means and is worried that the patient will require indefinite mechanical ventilation. Another option would be the use of noninvasive ventilation with BiPAP (i.e., nasal positive airway pressure during inspiration and expiration).

  • How would you respond to his wife's concern about the need for indefinite mechanical ventilation?

    The patient's sputum culture grows Haemophilus influenzae that is resistant to ampicillin. His antibiotics are changed, and 4 days later he is successfully extubated. After 14 days in the hospital, he is ready to be discharged.

  • After the patient is discharged, how would you provide follow-up, and what are your treatment options now?

Case Discussion

  • What tests and studies would you order in this patient?

    A chest radiographic study should be obtained. Although the value of a routine chest radiographic study in patients with an exacerbation of COPD has been debated, this patient has a productive cough, low-grade temperature, and localizing chest pain, all of which indicate the existence of an intrathoracic abnormality, stressing the importance of a chest radiograph.

    Despite a pulse oximetry reading of 91%, arterial blood gas measurements are indicated for in this patient. There are several factors that can cause a poor correlation between the pulse oximetry value and the PaO2, as measured by arterial blood gas determinations. It is poor in patients with jaundice or dark skin pigmentation, as well as in those with poor peripheral circulation. Furthermore, under various physiologic and pathologic conditions (e.g., changes in the pH or 2,3-diphosphoglyceric acid level), the oxyhemoglobin dissociation curve can be shifted to the right or left. Therefore, the oximeter can either underpredict or overpredict the actual PaO2. Finally, in a patient with a moderately severe pulmonary process, knowledge of the PaCO2 and pH is imperative.

    A complete blood count and chemistry panel should be obtained. The complete blood count can provide useful information regarding the severity of the infectious process (e.g., leukocytosis). Furthermore, significant polycythemia may indicate the existence of long-standing hypoxia, which signifies the chronicity and severity of the disease. The chemistry profile can provide valuable information concerning electrolyte imbalance (e.g., hyponatremia in the syndrome of inappropriate antidiuretic hormone secretion) or volume depletion. Knowledge of the serum bicarbonate level is useful in conjunction with the arterial blood gas findings to assess the chronicity of any respiratory acid base disorders.

    P.351


    In a patient of advanced age with risk factors for coronary artery disease (tobacco abuse and hypertension) and chest pain, an electrocardiography is indicated. Furthermore, many types of arrhythmias (e.g., multifocal atrial tachycardia) are seen predominantly in the setting of decompensated pulmonary disease.

    The theophylline level must be determined. Because this drug has a narrow therapeutic index, close monitoring of the serum levels is essential in acutely ill patients.

  • What is your diagnosis based on the information you have, and how would you manage this patient?

    The patient has a right middle lobe pneumonia and, as a result, an exacerbation of his COPD. Given the lack of cough and sputum production in his past history, as well as the bullae noted in the chest radiographic study, his clinical picture is consistent with emphysema, as opposed to chronic bronchitis. Most patients have a combination of both disorders. He should be admitted to the hospital.

  • What therapy should you institute while the patient is in the hospital?

    Blood and sputum cultures should be obtained. A Gram's-stained sputum specimen should be examined both by the primary physicians and by the laboratory technician.

    Inhaled -adrenergic agonists (e.g., 0.5 mL of albuterol in 1.5 mL of saline) are the mainstay of treatment for a COPD exacerbation. The initial dosing frequency of this medication depends on the severity of the disease; it can be administered every 1 to 3 hours. As the patient's condition improves, the dosing frequency can be reduced to every 4 to 6 hours. Although metered-dose inhalers can be used with a similar degree of success, their efficacy depends on the ability of the patient to coordinate the timing of the inhalation and the activation of the inhaler, making them a less-than-optimal tool in an acutely ill patient.

    The role of theophylline in the management of an acute exacerbation of COPD remains controversial. Most authorities agree that theophylline is a weak bronchodilator with a low therapeutic index. In a randomized, controlled study, the addition of aminophylline to a well formulated therapeutic regimen in hospitalized patients with COPD failed to show any benefit in terms of improvement in lung function or on the dyspnea scale. If used, theophylline levels should be monitored closely and the patient observed for any signs or symptoms of toxicity.

    Inhaled anticholinergics may also be useful. Ipratropium bromide is the agent of choice. It is available in a metered-dose inhaler formulation, and can be given in-line into ventilator tubing. Occasional blurred vision or urinary retention has been noted in patients using it. For the treatment of a patient with stable COPD, tiotropium bromide is superior to -agonists. A combination of the two can modestly improve the bronchodilation achieved, as well as prolong the effective duration of action of each agent. The starting dose is one inhalation twice daily.

    The use of systemic corticosteroids, like that of theophylline, is controversial. A stronger case for their use can be made if the patient has exhibited a previously documented steroid response, has eosinophilia, or has shown a significant bronchodilator response to the inhaled agents. A reasonable starting dose is 40 to

    P.352


    60 mg of intravenous methylprednisolone every 6 hours. This regimen is changed to an oral form (e.g., prednisone, 40 to 60 mg per day) with rapid tapering. Monitoring of the side effects (e.g., hyperglycemia, mental status changes, and gastritis) is crucial. Inhaled corticosteroids have no role in the acute management of this patient.

    Antibiotics are indicated even when an infiltrate is not found on the chest radiograph. In a well-designed, controlled clinical trial, patients with COPD treated with broad-spectrum oral antibiotics (the newer cephalosporins, macrolides, and fluoroquinolones) did better than the control patients. In this situation, the choice of antibiotic depends on the Gram's stain findings of the sputum. Bacteria commonly responsible for lower respiratory tract infections in this patient population include Streptococcus pneumoniae, H. influenzae, and Branhamella catarrhalis. The latter two usually produce -lactamase.

  • How would you respond to his wife's concern about the need for indefinite mechanical ventilation?

    The condition of patients with COPD frequently deteriorates to the point where they require ventilatory support. The prognosis for weaning the patient from the ventilator, as well as the future quality of life, depends on the patient's premorbid lung function and functional state. Although the status of this patient's pulmonary function is unknown, given his high quality of life and functional status before this episode, the odds are overwhelmingly in his favor that he can be successfully extubated. Therefore, very aggressive treatment is indicated.

  • After the patient is discharged, how would you provide follow-up, and what are your treatment options now?

    Every attempt should be made to encourage him to stop smoking. The rate of pulmonary function loss, in smokers who quit smoking, reverts gradually toward the rate seen in the normal population. The risk of lung cancer and heart disease also declines significantly.

    Among the therapeutic interventions now available are well-designed smoking cessation therapy groups, such as the one offered by the American Lung Association, as well as the supervised use of nicotine gum. It is also important that a follow-up chest radiographic study be obtained within 4 to 6 weeks to demonstrate disappearance of the infiltrate. An unresolved infiltrate could be due to lung cancer (especially with the volume loss seen earlier on his chest radiograph). The incidence of lung cancer is much higher in smokers with COPD than in those without. This risk also diminishes significantly with the cessation of smoking. The patient should also receive annual influenza vaccines and, although a controversial measure, a one-time polyvalent pneumococcal vaccine. His medications should include an inhaled agonist or ipratropium bromide (Atrovent), or both. The judicious use of sustained-dose oral theophyllines and steroids (inhaled if possible) may be indicated. Finally, a repeat arterial blood gas measurement on room air should be performed when the patient's condition is clinically stable. Supplemental oxygen for patients with a PaO2 of 55 mm Hg or less can improve a patient's cognitive ability, exercise tolerance, and right heart function, as well as prevent the development of pulmonary hypertension. Ultimately, it can lengthen the patient's life span.

P.353


Suggested Readings

Chronic obstructive pulmonary disease: disorder of the cardiovascular and respiratory systems. Proc Am Thorac Soc 2005;2:1 94.

Snow V, Lasher S, Mottur-Pilson C. Evidence for management of acute exacerbations of chronic obstructive pulmonary disease. Ann Int Med 2001;134:595 599.

Idiopathic Pulmonary Fibrosis

  • What are the basic pathologic events that lead to interstitial lung disease (ILD)?

  • What are the typical pulmonary function test (PFT) abnormalities observed in the setting of idiopathic pulmonary fibrosis (IPF)?

  • What is the outcome if IPF is left untreated, and is the diagnosis one of exclusion?

  • What are the presenting symptoms of IPF?

Discussion

  • What are the basic pathologic events that lead to ILD?

    Regardless of the underlying cause of ILD, the morphologic pattern of progression is similar. A known or unknown stimulus or immunologic event causes alveolar epithelial/endothelial injury resulting in migration of inflammatory cells to the alveolar structures. Unlike acute insults, as seen in bacterial pneumonia (which results in a transient inflammatory infiltrate), the injury of ILD is persistent or repetitive. The persistence of the injury and inflammation damages the parenchymal cells and causes disruption of the alveolar capillary membrane. Inflammation and abnormal repair then lead to mesenchymal cell proliferation (fibroblasts), with the attendant production of excess collagen and connective tissue elements expanding the extracellular matrix. Ultimately, the normal architecture of the lung is replaced by fibrotic bands and cystic spaces known as honeycomb lung. IPF is the prototypic ILD whose underlying pathologic process is usually interstitial pneumonia.

  • What are the typical PFT abnormalities observed in the setting of IPF?

    The characteristic PFT abnormalities include a gradual reduction of lung volumes and airflow (FEV1 and FVC), with preservation of the FEV1/FVC ratio. The FEV1/FVC can be normal or increased due to the increased elastic recoil of the stiff lung parenchyma. Patients with some ILD (e.g., chronic hypersensitivity pneumonitis, Langerhans cell granulomatosis of the lung, and cystic fibrosis) may initially exhibit normal or increased lung volumes secondary to small airway involvement with airflow obstruction (a reduced FEV1/FVC ratio) and air trapping. In addition, the development of emphysema in conjunction with any type of ILD may be initially associated with relatively mild abnormalities on routine PFTs. It is, therefore, important to remember

    P.354


    that the absence of typical PFT abnormalities does not exclude IPF or any other ILD. The gas exchange at rest is initially normal in many patients with IPF; however, exercise-induced desaturation is one of the earliest signs and the most sensitive means to detect the disease. The diffusion capacity is typically reduced but may be normal, especially in the early stages of the disease.

  • What is the outcome if IPF is left untreated, and is the diagnosis one of exclusion?

    IPF steadily progresses even when treated. With time, the chronic inflammatory response (alveolitis) produces fibrosis, along with the classic physiologic abnormalities already described. Although the etiology is unknown, the clinicopathologic manifestations are specific, and, therefore, it is not a diagnosis of exclusion. Up to 50% of patients die between 2 and 3 years after diagnosis. At present only lung transplantation is a viable treatment in eligible subjects. Currently, novel biologic agents are being tested.

  • What are the presenting symptoms of IPF?

    The insidious onset of breathlessness and nonproductive cough is common to most cases. Fatigue, low-grade feverishness, arthralgias, and myalgias are also relatively common, but nonspecific, symptoms. The occurrence of frank arthritis, myositis (muscle tenderness and weakness), photosensitivity, Raynaud's phenomenon, visual problems, and so on, suggests the existence of other systemic processes, such as collagen vascular disease, vasculitis, or sarcoidosis. Dry inspiratory crackles may be the only physical finding, although digital clubbing is seen in 40% to 70% of cases. A chest radiograph usually reveals reticular (linear) or reticulonodular opacities in the lower lung zones. The high-resolution CT scan indicates peripheral and basilar interstitial infiltrates and honeycombing.

Case

A 58-year-old woman is referred for the evaluation of breathlessness and cough. She first began noticing dyspnea on exertion approximately 3 to 4 years ago when using her floor sweeper. However, she noted no limitation when performing any of her other usual activities. Her dyspnea worsened slightly over the ensuing years without any other symptoms until 9 months ago, when a nonproductive cough developed. This was treated with antibiotics and inhaled bronchodilators, without improvement. Over the last 9 months, her breathlessness has worsened and she now has trouble climbing one flight of stairs. She tires easily and occasionally feels feverish, but has not experienced arthralgias, myalgias, night sweats, or other constitutional symptoms. She has experienced no chest pain or hemoptysis and has no history of cardiopulmonary disease. Her past medical history is unremarkable. Medications include inhaled bronchodilators. She is married and has never smoked. She has worked as a retail sales clerk for 17 years without exposures. She has had no pet birds, leaky pipes, or moldy conditions in her home.

Physical examination reveals a respiratory rate of 20 unlabored breaths per minute with dry inspiratory rales heard over the lower third of the posterior lung fields. She has no clubbing or edema. Laboratory evaluation reveals a normal hemogram and biochemical

P.355


profile. Antinuclear antibodies show weak positivity at 1:80. Testing for rheumatoid factor is negative.

A chest radiograph shows reticular opacities that are most prominent in the lower lung zones as well as reduced lung volumes and the high-resolution computed tomography scan (HRCT) shows peripheral and basilar reticular infiltrates as well as honeycomb change in a similar distribution. PFTs reveal a total lung capacity of 70% of predicted and a functional residual capacity that is 66% of predicted. The FEV1 is 50% of predicted with an FVC that is 58% of predicted. The FEV1/FVC ratio is 88%. Her diffusion capacity is 65% of predicted. The PaO2 on room air while resting is 60 mm Hg, which drops to 38 mm Hg with exercise.

  • How would the diagnosis of IPF best be confirmed in this patient?

  • If the thoracoscopic lung biopsy specimen reveals the presence of the usual interstitial pneumonitis, what are the treatment options for this patient?

Case Discussion

  • How would the diagnosis of IPF best be confirmed in this patient?

    Although the clinical, radiologic, and physiologic picture in this patient is most suggestive of IPF, other interstitial lung processes, such as chronic hypersensitivity pneumonitis, nonspecific interstitial pneumonia (NSIP), asbestosis, stage III sarcoidosis, and collagen vascular disease, may present in an identical manner. However, the typical HRCT and lack of evidence for one of the aforementioned ILDs, establishes the diagnosis. In cases where the radiologic presentation is not typical, surgical lung biopsy by the video-assisted thoracoscope (VATS) is indicated for the diagnosis. The histologic expression of IPF is usual interstitial pneumonia (UIP). This is characterized by temporal heterogeneity in which end stage honeycomb is adjacent to normal lung and there is alveolar wall fibrosis of varying degrees. Another important feature is the presence of fibroblastic foci, which are subepithelial collections of myofibroblasts in loose connective tissue stroma.

  • If the thoracoscopic lung biopsy specimen reveals the presence of the usual interstitial pneumonitis, what are the treatment options for this patient?

    Corticosteroids and immunosuppressive drugs (azathioprine and cydophosphamide) have been recommended for IPF. However, at present there is no evidence that this treatment improves outcome. For eligible patients, transplantation is an option. The addition of N-acetylcysteine to prednisone azathioprine regimen shows promise for a few patients. Newer biologics that potentially inhibit fibroproliferation are in trials.

Suggested Readings

King TE. Idiopathic pulmonary fibrosis. In: Schwarz MI, King TE, eds. Interstitial lung disease, 4th ed. Toronto: BC Decker, 2003.

P.356


Pleural Disease

  • What is a pleural effusion?

  • What are the physical findings associated with a pleural effusion?

  • What is the significance of distinguishing between a transudative and an exudative pleural effusion?

  • What testing distinguishes between pleural transudates and exudates?

  • What is an empyema?

  • How do you develop a treatment plan in the patient with a pleural effusion?

Discussion

  • What is a pleural effusion?

    A pleural effusion is an abnormal collection of fluid in the potential space between the visceral and parietal pleura. Normally, this space contains only a few milliliters of fluid, which serves to lubricate these surfaces.

  • What are the physical findings associated with a pleural effusion?

    Pleural effusions can be detected on physical examination if they are of sufficient volume to produce a fluid level in the chest and compress underlying lung tissue. Dullness to percussion with decreased or absent breath sounds in a dependent anatomic location is typical of pleural effusion. Egophony is an important finding that distinguishes an effusion stemming from atelectasis secondary to bronchial obstruction. Lobar consolidation with a patent bronchus, as occurs in pneumonia, may be difficult to distinguish from an effusion, because these two abnormalities often coexist. Whispering pectoriloquy may be heard in the presence of a consolidation but is absent over a pleural effusion. In addition, physical findings that suggest a systemic illness, such as congestive heart failure, cirrhosis, and lupus erythematosus, provide important clues to the potential cause and nature of a pleural effusion discovered on physical examination or a radiographic study.

  • What is the significance of distinguishing between a transudative and an exudative pleural effusion?

    This division is an important first step in the diagnostic evaluation of a pleural effusion. In the context of a transudative pleural effusion, the pleura itself is not diseased but fluid is accumulating because of the effect of abnormal Starling forces stemming from a systemic illness, such as congestive heart failure, cirrhosis, or nephrotic syndrome. In these conditions, pleural fluid accumulates for the same reasons that peripheral edema and ascites develop. In the setting of an exudative effusion, the pleura is primarily involved by the disease. Examples include malignancies in the pleura (usually metastatic), infections, collagen vascular diseases, and pulmonary infarctions. In these conditions, the pleural surface is injured and fluid accumulates independent of Starling forces.

  • P.357


  • What testing distinguishes between pleural transudates and exudates?

    There are a number of simple laboratory tests that can help in distinguishing transudative and exudative pleural effusions. Exudates have a higher protein content and lactate dehydrogenase (LDH) level than transudates because the mesothelial cells are injured by the disease process. A pleural fluid protein level greater than 50% of the patient's corresponding plasma protein level, an LDH level greater than 180 IU, or an LDH level greater than 60% of the patient's corresponding plasma level distinguishes pleural exudates from transudates. If any of these are present, it is more than 90% likely that the effusion is an exudate. The finding of a high white blood cell count in the pleural fluid suggests the presence of an exudate but is a less reliable indicator than the protein and LDH values. Once an exudate has been identified, other studies can be performed to help elucidate its cause. These include cytologic analysis, microbiologic stains and cultures, pH determination, and serologic testing for collagen vascular disease.

  • What is an empyema?

    Empyema is an infection in the pleural space. This infection may be bacterial, mycobacterial, or fungal in origin. In addition to more common bacterial species, Actinomyces and Nocardia can also cause empyema. Empyemas must be distinguished from parapneumonic effusions. In this case, a pneumonia abutting the pleura can result in an inflammatory exudate. The diagnostic hallmark of empyema is identification of the causative organism either by bacterial staining or culture of the fluid. Often this is not possible due to prior antibiotic therapy. For example, a clinical diagnosis of empyema is made on the basis of overall clinical presentation. However, a low pleural glucose level (<50 mg/dL) or a pH (<7.30), or both, suggest the presence of empyema, but are also seen in malignant effusions, esophageal rupture, and rheumatoid arthritis. Empyemas are difficult infections to cure with antimicrobial therapy alone, particularly when bacterial in origin. These empyemas are essentially intrapleural abscesses and, like most abscesses, require drainage to achieve resolution.

  • How do you develop a treatment plan in the patient with a pleural effusion?

    Treatment depends on the etiology of the effusion. If it is a transudate, effective treatment of the congestive heart failure, nephrotic syndrome, or cirrhosis, if possible, often results in resolution. If the effusion is associated with an infection, such as a parapneumonic effusion or an empyema, definitive treatment of the infection is indicated. Malignant effusions must be addressed in the context of the underlying malignancy. Therefore, the treatment of a pleural effusion depends on the information gleaned during an appropriate clinical evaluation of the patient as a whole.

Case

A 43-year-old man with long-standing seropositive rheumatoid arthritis presents to the emergency room complaining of right pleuritic chest pain. He was started on prednisone

P.358


by his physician 1 week earlier for an acute flare-up of synovitis in his wrists and hands. For several days before the onset of the pleuritic pain, the patient noted malaise, anorexia, and fevers. The night before presentation, he noted the onset of sharp, nonradiating pain in the right chest, which worsened with coughing or deep breathing. His cough is nonproductive. He denies cigarette or alcohol use.

On physical examination, he is found to be in moderate distress because of his chest pain. His blood pressure is 120/70 mm Hg, pulse is 120 beats per minute and regular, the respiratory rate is 20 breaths per minute, and temperature is 38 C (100.4 F). His physical examination is remarkable for good dentition; a normal jugular venous pressure; a regular tachycardia without murmurs, gallops, or rubs; and no peripheral edema. Lung examination reveals dullness to percussion at the right base with absent breath sounds in that area. Egophony is present at the right base but whispering pectoriloquy is absent. The left lung is clear except for a small area of decreased breath sounds at the base.

A complete blood count reveals a mild normocytic anemia with a hemoglobin level of 12.5 g/dL. The white blood cell count is 13,000/mm3 with an increase in the number of band forms. A chest radiograph shows a moderate right pleural effusion, a small left pleural effusion, and a normal cardiac silhouette.

Thoracentesis of the right pleural effusion yields 250 mL of yellow, slightly cloudy fluid. The white blood cell count in the fluid is 3,500/mm3 with 90% neutrophils. The red blood cell count is 1,000/mm3. The protein level is 4.0 g/dL, the LDH level is 400 IU, the glucose content is 10 mg/dL, and the pH is 7.12.

  • What is the most likely cause of the right pleuritic chest pain?

  • What further tests would you do to verify your diagnosis?

  • How would you manage this patient's acute problem?

  • What are the intrathoracic manifestations of rheumatoid arthritis?

Case Discussion

  • What is the most likely cause of the right pleuritic chest pain?

    The presentation consisting of fever, acute pleuritic pain, and an exudative pleural effusion with a predominance of neutrophils is most consistent with a bacterial infection (empyema) of the pleural space, or empyema. Empyema is usually the result of a pneumonia that extends to and involves the adjacent pleura. The pleural fluid may have a low glucose level and usually has a low pH (<7.30). The most common causes include anaerobic bacteria (often resulting from an aspiration pneumonia), Staphylococcus aureus, pneumococcus, and tuberculosis. In immunocompromised hosts, the differential diagnosis includes fungi and other opportunistic pathogens.

    In this patient, the diagnosis of pulmonary embolism with subsequent pulmonary infarction cannot be excluded. This can present with fever and pleuritic pain as well. However, the pleural fluid is usually bloody as a result of local tissue infarction. In addition, hemoptysis may be an associated finding. This patient's effusion had a low red blood cell count and he had no risk factors for pulmonary emboli, such as prolonged bed rest or recent trauma.

    P.359


    Rheumatoid involvement of the pleura is common. Most patients with rheumatoid arthritis have a pleural effusion at some time during the course of the disease. The typical characteristics of the fluid are an exudative with a low glucose level and a high rheumatoid factor level. The presentation is subacute when the effusion becomes large enough to cause symptoms or, in most cases, is noted on examination or a radiographic study in an asymptomatic patient.

    Constrictive pericarditis may be a rare consequence of rheumatoid involvement of the pericardium. However, it is usually manifested by right-sided congestion with an elevated jugular venous pressure (usually with a Kussmaul's sign), hepatomegaly, and peripheral edema.

  • What further tests would you do to verify your diagnosis?

    Gram's staining and culture of the pleural fluid in a patient with empyema is important to identify a causative organism and direct the choice of antibiotic therapy. The Gram's stain findings are often positive and can narrow the differential diagnosis and the antibiotic choices. When tuberculous empyema is suspected, acid-fast staining of a sample of pleural fluid is indicated. Pleural tissue culture is optimal. Subsequent cultures and sensitivities are useful in adjusting drug therapy. In the event of empyema caused by anaerobic bacteria, the Gram's stain findings are more often than not negative, and cultures may yield negative results unless the fluid is carefully handled and processed anaerobically. When anaerobic empyema is suspected, empiric therapy is often necessary if an organism cannot be identified.

    A CT angiogram is indicated in cases of suspected pulmonary emboli and will show segmental or subsegmental filling defects in pulmonary arteries.

    Determination of the rheumatoid factor level in the pleural space can be performed to ascertain whether a pleural effusion in a patient with rheumatoid arthritis is related to the underlying rheumatoid process. However, the presence of a high rheumatoid factor does not exclude a secondary complication of a rheumatoid effusion such as infection.

    An echocardiogram is a useful way to assess the pericardium when pericardial disease, such as constrictive pericarditis, is suspected.

  • How would you manage this patient's acute problem?

    The therapy for empyema requires appropriate antibodies directed toward the known or presumed causative organism, or organisms. In patients with bacterial empyemas, traditional therapy also involves chest tube placement and, sometimes, surgical drainage of the pleural space because this essentially represents an abscess cavity and antibiotic therapy alone is usually ineffective. Drainage is not done in the event of tuberculous empyema, however. The therapy for this is prolonged (6 to 12 months) treatment with antituberculous drugs. Some clinicians have recommended that, in the event of pneumococcal empyema, antibiotic therapy alone is often effective when the fluid is not loculated and the patient is clinically doing well. In this patient, the low pH of the fluid and significant pleuritic pain would prompt placement of a small percutaneous catheter into the right pleural space to drain the empyema completely. Pericardiocentesis is the treatment for pericardial tamponade, but is not indicated for constrictive pericarditis. Prednisone can be administered to

    P.360


    suppress a flare-up of rheumatoid disease involving the joints, although it does not appear to have any effect on pleural or pericardial involvement.

  • What are the intrathoracic manifestations of rheumatoid arthritis?

    Rheumatoid arthritis is a systemic illness. As already noted, most patients will have pleural involvement. Pericardial involvement is less common and, fortunately, is rarely clinically significant. Pulmonary nodules can also form, particularly in patients with rheumatoid nodules on their extremities. These nodules are similar histologically to the peripheral nodules, they wax and wane with the intensity of the systemic disease, and are rarely significant clinically. In addition to these manifestations, rheumatoid disease can cause an ILD either usual interstitial pneumonia, NSIP, organizing pneumonia, or acute interstitial pneumonia. Rarely, bronchiolitis obliterans may occur in rheumatoid arthritis. This is characterized by a progressive, irreversible illness similar to emphysema in its radiographic appearance and physiologic abnormalities, and is usually fatal within 5 years of presentation.

Suggested Readings

Hamm H, Light RW. Parapneumonic effusion and empyema. Eur Respir J 1997;10:1150 1156.

Pulmonary Complications of Human Immunodeficiency Virus Infection

  • What are the pulmonary complications in a patient with human immunodeficiency virus (HIV) infection?

  • What tests would help you establish a specific diagnosis?

Discussion

  • What are the pulmonary complications in a patient with HIV infection?

    HIV is a lymphotropic retrovirus that infects T4 (helper) lymphocytes, B cells, and monocytes, leading to a defect in cell-mediated immunity, which then predisposes to the development of a variety of neoplasms and opportunistic infections. The lung is one of the primary target organs in HIV disease, and pulmonary complications are the leading cause of hospitalization and death in HIV-infected patients. The spectrum of pulmonary disorders associated with HIV infection includes both infectious and noninfectious diseases. The infectious causes of pulmonary disease include both opportunistic and nonopportunistic agents. The most common opportunistic organisms are Pneumocystis jiroveci, cytomegalovirus, and Mycobacterium avium-intracellulare (MAI). Although opportunistic infections are common in HIV-infected patients, these patients are also more susceptible to nonopportunistic infections,

    P.361


    including pyogenic organisms (S. pneumoniae and H. influenzae), Mycobacterium tuberculosis, and fungal infections. The specific infection the patient acquires depends on the degree of immune deficiency and his or her exposure to specific organisms. The noninfectious pulmonary disorders include Kaposi's sarcoma, non Hodgkin's lymphoma, lymphocytic interstitial pneumonitis, nonspecific interstitial pneumonia, alveolar proteinosis, bronchiolitis obliterans organizing pneumonia, primary pulmonary hypertension, and emphysema.

  • What tests would help you establish a specific diagnosis?

    Chest imaging, arterial blood gas determinations, a complete blood count, serum LDH measurement, and sputum studies should all be obtained in an HIV-infected patient presenting with fever and increasing shortness of breath. Although the chest radiograph or CT scan can be helpful, the radiographic manifestations of pulmonary disease overlap significantly in this group of patients. Most patients with Pneumocystis pneumonia (PCP) are hypoxic and hypocapnic, and exhibit a widened alveolar-arterial gradient, often before any abnormality is detected on a chest radiograph. The complete blood count in HIV-infected patients typically demonstrates an absolute lymphopenia, which primarily stems from a decrease in the number of T4 cells. The LDH level is elevated in 95% of the patients and has been shown to increase with worsening symptoms and decline in response to therapy. Sputum in patients with pyogenic bacterial infections is often purulent, and the Gram's stain and culture findings should dictate the antibiotic choice. Patients with PCP often have a nonproductive cough, making it necessary to obtain a sputum specimen that is induced by the inhalation of hypertonic saline. Sputum and blood cultures are also indicated for MAI infection and tuberculosis. This has a yield of 25% to 85%, depending on the experience of the person performing the test. Methenamine silver staining is used to identify the cysts. If sputum studies are unrevealing, staining of bronchoalveolar cells (BAL) has a higher positivity rate.

Case

A 37-year-old homosexual man, known to be HIV positive, is seen for evaluation of progressive dyspnea and fever. He was well until 7 to 8 months ago, when he noted the onset of weight loss, diffuse adenopathy, and night sweats. Over the last month, he has noted progressive dyspnea, a dry, nonproductive cough, and daily fever spikes. He has smoked 20 cigarettes a day from the age of 18 years, denies alcohol or drug abuse, and has lived in the Ohio River Valley as well as the Southwest. Physical examination reveals a blood pressure of 110/65 mm Hg, pulse of 100 beats per minute, respiratory rate of 32 breaths per minute, and temperature of 39 C (102.2 F). Throat examination findings are remarkable for a white exudate on the posterior pharyngeal wall. The lymph nodes are diffusely enlarged and nontender. During chest examination, bilateral crackles are noted. The remainder of the examination findings are unremarkable. A chest radiograph reveals the presence of diffuse bilateral interstitial infiltrates.

  • What is the differential diagnosis in this patient, and do the chest radiograph findings influence this?

  • P.362


  • If the initial test results do not confirm your diagnosis, what test would you do next?

  • What therapy would you initiate, and is there a role for prophylactic therapy?

Case Discussion

  • What is the differential diagnosis in this patient, and do the chest radiograph findings influence this?

    Although malignancy and a chronic infectious process such as tuberculosis could account for these chronic and subacute symptoms, this patient's clinical picture is most consistent with a complication of HIV infection. Besides the spectrum of pulmonary disorders associated with HIV infections, primary cardiac disorders should also be considered when dyspnea and bilateral rales are encountered. The fever and chills in this patient indicate an infectious cause. The differential diagnosis in this patient would include pneumocystis, tuberculosis, and infection with nonopportunistic pulmonary pathogens, such as S. pneumoniae, H. influenzae, and S. aureus.

    As already mentioned, the chest radiograph can be helpful in making the diagnosis, although there is significant overlap in the radiographic manifestations of the various pulmonary diseases. In PCP, the most common finding is a diffuse increase in the interstitial and alveolar markings, although nodular infiltrates, cavities, pneumatoceles and pneumothoraces, and pleural effusions have all been observed in this setting. In addition, 20% of patients with PCP can have a normal radiograph. Kaposi's sarcoma often presents with nodular infiltrates, with or without a pleural effusion. The effusion is either serosanguineous or hemorrhagic and is due to pleural involvement by the sarcoma. The presence of intrathoracic adenopathy suggests tuberculosis, non Hodgkin's lymphoma, Kaposi's sarcoma, or MAI infection.

  • If the initial test results do not confirm your diagnosis, what test would you do next?

    If the sputum specimen findings are nondiagnostic, the next diagnostic procedure would be fiberoptic bronchoscopy with bronchoalveolar lavage and, in some cases, a transbronchial biopsy. This allows the alveolar tissue to be directly sampled. Bronchoalveolar lavage is performed by placing the bronchoscope in the distal airway, instilling 100 to 200 mL of saline into the airway, and then immediately removing the solution. The lavage technique has a yield of 75% to 95% in the setting of PCP, and transbronchial biopsy has a yield of 85% to 95%. Kaposi's sarcoma, however, is difficult to diagnose using bronchoscopy. If bronchoscopic findings are nondiagnostic, an open lung biopsy should be done. This involves an open thoracotomy and is rarely needed.

  • What therapy would you initiate, and is there a role for prophylactic therapy?

    Treatment with trimethoprim sulfamethoxazole should be started in conjunction with intravenous corticosteroids. The role of prophylactic therapy is under investigation. It also appears that prophylactic oral therapy with trimethoprim sulfamethoxazole is highly efficacious in preventing recurrence of PCP. However, many patients are unable to endure prolonged therapy because of adverse side effects. Aerosolized pentamidine is also used for PCP prophylactic treatment, and has been shown to be effective.

P.363


Suggested Readings

Hopewell PC, Luce JM. Pulmonary manifestations of the acquired immunodeficiency syndrome. Clin Immunol Allergy 1986;6:489.

Murray JF, Mills J. Pulmonary infectious complications of human immunodeficiency virus infection: part I. Am Rev Respir Dis 1990;141:1356.

Murray JF, Mills J. Pulmonary infectious complications of human immunodeficiency virus infection: part II. Am Rev Respir Dis 1990;141:1582.

Solitary Pulmonary Nodule

  • What is a solitary pulmonary nodule (SPN)?

  • What percentage of SPNs is benign?

  • What clinical and radiologic findings are associated with a higher incidence of malignancy in an SPN?

  • What is the most common cause of an SPN?

Discussion

  • What is an SPN?

    Solitary pulmonary nodules are single opacities located entirely within the lung parenchyma and usually less than 4 cm in diameter. They are not associated with atelectasis or hilar adenopathy on plain chest roentgenograms.

  • What percentage of SPNs is benign?

    Seventy-five to 85% of SPNs are benign, and 15% to 25% are malignant (either primary or metastatic disease). The physician's role is to expedite the workup and resection of potentially curable malignant SPNs, while avoiding costly evaluations and painful thoracotomies for SPNs that are benign or already unresectable (i.e., metastatic).

  • What clinical and radiologic findings are associated with a higher incidence of malignancy in an SPN?

    There are several features that suggest malignancy:

    • Age. Most SPNs in adults younger than 35 years are benign. The risk of malignant disease increases with increasing age.

    • Nodule size. More than 80% of the SPNs larger than 3 cm in diameter are malignant; 20% or fewer of the SPNs less than 2 cm in diameter are malignant. Spiculation on CT scan of the chest is more likely malignant.

    • Presence and pattern of calcification. Calcification, particularly that with a central, laminated, or diffuse pattern, is suggestive of benign disease. Malignant disease only rarely shows evidence of calcification, and more frequently exhibits an eccentric pattern.

    • History of prior malignancy. As many as 30% of malignant SPNs are metastases from extrathoracic malignancies.

    • Smoking history. Although the effect of smoking on malignancy in the setting of SPNs has not been specifically determined, there is a

      P.364


      well-known association between smoking and the development of primary bronchogenic carcinoma. This risk is similar to that of the general nonsmoking population between 10 and 15 years after smoking cessation.

  • What is the most common cause of an SPN?

    More than half of all SPNs are found to be granulomas on pathologic examination. Hamartomas represent the next most common benign cause, but constitute less than 10% of all SPNs.

Case

A 40-year-old woman is seen for a preoperative evaluation before undergoing laparoscopic cholecystectomy. Her medical history is remarkable for mild untreated hypertension. She has undergone no previous surgical procedures. She had a 5-pack-year history of smoking, but quit 8 years ago. She was born and raised in Cincinnati. She works as a paralegal in a downtown law firm. She takes no medications other than an occasional aspirin for headache.

A thorough review reveals symptoms referable to her cholelithiasis. Specifically, she denies any systemic or chest complaints, including fever, malaise, weight change, myalgias or arthralgias, chest pain, shortness of breath or dyspnea on exertion, cough, and hemoptysis.

Physical examination reveals a mildly obese woman in no distress. Vital signs are normal except for a blood pressure of 170/90 mm Hg. Head and neck findings are normal, and her lungs are clear. Heart findings are also normal. Abdominal examination reveals right upper quadrant tenderness in response to deep palpation without rebound or guarding. There are no masses or hepatosplenomegaly. Stool is guaiac negative. Her extremities are normal, as are the findings from a thorough neurologic examination. There is no adenopathy.

Laboratory examination findings, including a complete blood count, routine chemistries, arterial blood gases measurement, and urinalysis, are within normal limits. An electrocardiogram is normal. A chest radiographic study reveals a 1.8-cm, round opacity in the left lower lobe, but is otherwise normal.

  • What is the most important next diagnostic step at this time?

  • What are the major possible diagnoses of this patient's nodule?

  • What noninvasive diagnostic test may help in distinguishing between the possible diagnoses in this patient?

  • If the test results up to this point have been nondiagnostic or indeterminate, what options should be presented to the patient at this time?

Case Discussion

  • What is the most important next diagnostic step at this time?

    If possible, an old chest radiographic study should be obtained. If the nodule was present and is unchanged in size on a study from at least 2 years before, it is very likely that this represents a benign lesion, and no further workup is necessary. Malignant lesions usually have a doubling time of weeks to months. In other words, a lesion that grows either very rapidly (days) or very slowly (years) is likely to be benign.

    P.365


    In the event of very rapid growth, the patient usually has other pulmonary symptoms consistent with a benign diagnosis, such as infection or pulmonary infarction.

  • What are the major possible diagnoses of this patient's nodule?

    The differential diagnosis for this patient includes granulomatous disease, bronchogenic carcinoma, hamartoma, pulmonary metastasis from an unknown primary tumor, and round pneumonia.

    Infectious granulomatous diseases resulting in SPNs include histoplasmosis, coccidioidomycosis, and tuberculosis. Granulomas can also appear as SPNs in the settings of sarcoidosis, rheumatoid arthritis, and vasculitides such as Wegener's granulomatosis. Primary bronchogenic carcinoma is the most frequent source of resected malignant SPNs. Metastatic disease, often originating from primary adenocarcinomas of the breast, prostate, or colon, frequently present as SPNs. Round pneumonia is an uncommon presentation of an acute pulmonary infection in which the alveolar space-filling disease assumes a more rounded, nodular appearance. In the absence of other signs or symptoms, this can be confused with an SPN. Less common causes of SPNs include arteriovenous malformations, bronchogenic cysts, pulmonary infarction, and parasitic disease.

  • What noninvasive diagnostic test may help distinguish between the possible diagnoses in this patient?

    HRCT is indicated for this patient. SPNs are often found to be multiple on CT, suggesting the presence of either granulomatous disease or pulmonary metastases. Less than 1% of primary lung cancers present as multiple and synchronous lesions. CT is also very sensitive in defining the density and configuration of an SPN. The finding of a fat density in the nodule strongly suggests the diagnosis of hamartoma. Central, laminated, or diffuse patterns of calcification also suggest a benign diagnosis (particularly granulomatous disease or hamartoma), whereas eccentric calcification can be found in either benign or malignant disease. Less helpful is the configuration of the SPN. Poorly marginated or spiculated nodules are often malignant, but well-marginated spherical nodules can be either benign or malignant. Ipsilateral mediastinal or hilar adenopathy (defined by most radiologists as lymph nodes >1 cm in transverse diameter) can be associated with either benign or malignant lesions. However, adenopathy involving the hemithorax contralateral to the SPN is highly suggestive of nonresectable malignant disease.

  • If the test results up to this point have been nondiagnostic or indeterminate, what options should be presented to the patient at this time?

    There are three options at this point.

    Observation is appropriate in many patients, particularly those with a very low likelihood of malignancy or those for whom an invasive diagnostic procedure would carry an unacceptably high risk of morbidity and mortality. The course of the SPN can be monitored with serial imaging every 3 months for the first year, and every 6 months for the second year.

    Biopsy can be performed using either CT or fluoroscopy-guided transthoracic fine-needle aspiration (FNA) or fiberoptic bronchoscopy with transbronchial biopsy. The latter procedure is associated with a lower diagnostic yield, particularly for small (<2 cm) peripheral SPNs. In any event, if the diagnosis is not established, more

    P.366


    aggressive attempts to obtain definitive tissue must be pursued. Nondiagnostic tissue findings should not be construed as evidence of a benign lesion.

    Surgical lung biopsy is a third option. This has the advantage of being both a diagnostic and a therapeutic procedure. It is also associated with higher morbidity. Many surgeons perform mediastinoscopic lymph node biopsy before open thoracotomy, especially in cases of CT-proven mediastinal adenopathy, to avoid the more extensive procedure if possible.

Suggested Readings

Lillington GA, Caskey CI. Evaluation and management of solitary and multiple pulmonary nodules. Clin Chest Med 1993;14:111.

Midthun DE, Swensen SJ, Jett JR. Clinical strategies for solitary pulmonary nodule. Annu Rev Med 1992;43:195.

Midthun DE, Swensen SJ, Jett JR. Approach to the solitary pulmonary nodule. Mayo Clin Proc 1993;68:378.

Webb WR. Radiologic evaluation of the solitary pulmonary nodule. AJR Am J Roentgenol 1990;154:701.

Acute Pulmonary Embolism

  • What is a pulmonary embolism?

  • What are the common sources of pulmonary emboli?

  • What are the risk factors for pulmonary emboli?

  • Are all acute pulmonary emboli similar?

Discussion

  • What is a pulmonary embolism?

    A pulmonary embolism results from the migration of venous thrombi from the systemic veins to pulmonary arterial system, resulting in varying degrees of obstruction of pulmonary arterial blood flow. The incidence of pulmonary emboli in the United States exceeds 500,000 per year, with a mortality approaching 10%. If not diagnosed or if improperly treated, the mortality rate can reach 30%.

  • What are the common sources of pulmonary emboli?

    Up to 90% of pulmonary emboli originate from the deep venous system of the legs. The upper extremities can also be a source of venous thrombi. Usually related to trauma, congenital fibromuscular bands, or the use of central venous catheters, 12% of all upper extremity thrombi result in pulmonary emboli. In addition, blood clot formation in the pelvic veins may cause either septic or bland pulmonary emboli, especially in the setting of complicated obstetric procedures or gynecologic surgery.

    Other causes of pulmonary arterial obstructive emboli include air introduced during intravenous injections, hemodialysis, or the placement

    P.367


    of central venous catheters; amniotic fluid secondary to vigorous uterine contractions; fat as a result of multiple long bone fractures; parasites; tumor cells; or injected foreign material (talc, mercury).

    Table 8-2 Risk Factors for Venous Thrombosis

    Stasis Hypercoagulability Endothelial Injury
    Congestive heart failure Deficiency of antithrombin III Extensive pelvic surgery
    Obesity Deficiency of proteins C and S Prior injury
    Prolonged bed rest Malignancies Trauma
    Prolonged travel Oral contraceptives  
      Presence of a lupus anticoagulant  
      Factor V Leiden deficiency  

  • What are the risk factors for pulmonary emboli?

    Three basic risk factors, known collectively as Virchow's triad, are associated with thrombus formation and subsequent pulmonary emboli: stasis, hypercoagulability, and endothelial injury. Most clinical risk factors are derived from one of these pathogenic mechanisms, and these are listed in Table 8-2.

  • Are all acute pulmonary emboli similar?

    Pulmonary emboli produce several clinical syndromes. The rarest, acute massive occlusion, is defined as an embolus that occludes enough of the pulmonary circulation to produce circulatory collapse. In patients who do not survive this event, autopsy reveals occlusion, usually at the bifurcation of the main pulmonary artery, and the formation of saddle emboli. Pulmonary infarction refers to an embolism that obstructs enough blood flow to a portion of the lung, causing loss of viability of the lung tissue. This occurs in 10% of cases of acute pulmonary embolism. The third and most common clinical occurrence is pulmonary embolism without infarction. These are the most difficult to diagnose because they mimic other pulmonary and cardiac conditions. Because most emboli are multiple, both infarcted and noninfarcted areas in the lung can coexist.

Case

A 27-year-old woman presents to the emergency room after 24 hours of right-sided chest pain, which is worse with inspiration. She is short of breath and anxious. The patient denies sputum production, hemoptysis, cough or wheezing but states that she felt warm at home but did not take her temperature. She denies any recent injury or swelling of her legs, and is a very active person. The patient has no prior history of lung or heart disease.

She takes oral contraceptives, and has no known drug allergies. She has undergone no surgical procedures.

P.368


She smokes one pack of cigarettes per day, and does not consume alcohol. She denies intravenous drug use and has no risk factors for HIV disease. She works as an accountant. Her family history is negative for asthma and heart disease.

Physical examination reveals a mildly obese woman in moderate respiratory distress. Her temperature is 38.0 C (100.4 F), her pulse is 115 beats per minute, her blood pressure is 140/80 mm Hg, and her respiratory rate is 26 breaths per minute. No jugular venous distention is observed. Her chest is clear.

Cardiac examination reveals regular rate and rhythm, with normal intensity of the first and second heart sounds. There are no third or fourth sounds, murmurs, or rubs. Abdominal examination reveals positive bowel sounds and no hepatosplenomegaly. Her extremities show no cyanosis, clubbing, or edema.

Her laboratory values are as follows: hemoglobin, 14.5 g/dL; hematocrit, 42%; white blood cells, 6,000/mm3 with 74% segmented neutrophils and 26% lymphocytes. Peak expiratory flow is 450 L per minute, which is normal.

A chest radiographic study reveals a normal cardiac silhouette and clear lung fields, except for a small peripheral infiltrate in the lower left lobe. An electrocardiogram shows sinus tachycardia without ischemic changes. Arterial blood gas measurement performed on room air reveals a pH of 7.49, a PCO2 of 32 mm Hg, a PO2 of 60 mm Hg, and an alveolar-arterial oxygen gradient of 40 mm Hg.

  • What is the differential diagnosis?

  • What additional tests should be done to help narrow the differential diagnosis?

  • How do you interpret the additional test results?

  • What is the next step in diagnosing an acute pulmonary embolism?

  • What is the acute management of pulmonary embolism?

  • How long should anticoagulation therapy be continued?

  • What role would thrombolytic therapy have in this patient?

  • When should a vena caval filter be placed?

Case Discussion

  • What is the differential diagnosis?

    The differential diagnosis in this young woman with acute onset of shortness of breath and chest pain is lengthy. Not all breathing disorders are due to pulmonary disease because ischemic cardiac disease can present with dyspnea when associated with left ventricular failure. However, the nature and location of the pain, the lack of substantial cardiac risk factors, and the patient's age make cardiac ischemia unlikely.

    Several pulmonary disorders can have a similar presentation. Patients with acute bacterial pneumonia complain of shortness of breath, low-grade fever, and chest pain. However, pneumonia also usually causes sputum production and an elevated white blood cell count, which were not present in this patient. Asthma can also present insidiously with acute shortness of breath. However, the lack of a history of asthma, exposure to known triggers of asthma, and a normal peak expiratory flow make this diagnosis unlikely. A spontaneous pneumothorax can cause symptoms, yet it would be unusual for this to be accompanied by a low-grade fever.

    P.369


    The most common symptoms of pulmonary emboli include shortness of breath, pleuritic pain, cough, and hemoptysis. Pulmonary emboli should always be considered in a patient with acute shortness of breath and a known risk factor for thrombosis (oral contraceptives). Pleuritic chest pain, as seen in this patient, and hemoptysis occur only if the embolism causes a pulmonary infarction. Fevers as high as 39 C (102.2 F) have also been reported in the setting of infarction or a concurrent infection. On physical examination, the most common sign is isolated sinus tachycardia; however, in those patients with massive embolism, evidence of acute right ventricular failure may be found. When considering pulmonary embolism, there are no universal clinical findings and the absence of specific findings does not exclude the diagnosis.

  • What additional tests should be done to help narrow the differential diagnosis?

    To help differentiate between the various diagnoses, a chest radiographic study, electrocardiogram, arterial blood gas analysis, and Gram's staining of a sputum sample should be done.

  • How do you interpret the additional test results?

    In the setting of an acute pulmonary embolism, the arterial blood gas measurement classically reveals a low PCO2, low PO2, and a widened alveolar-arterial oxygen gradient. However, many other disorders cause similar abnormal arterial blood gas results and 10% to 15% of patients with proven pulmonary emboli maintain a normal alveolar-arterial oxygen gradient.

    The chest radiograph findings of pulmonary emboli are nonspecific. Typically, infiltrates, atelectasis, effusions, or any combination of these are encountered. It is not usual for the chest radiograph to be normal. A peripheral wedge-shaped infiltrate, sometimes referred to as a Hampton's hump, occurs when the embolism is associated with infarction, and occasionally decreased pulmonary vascular markings are noted (Westermark's sign), indicative of decreased blood flow to a section of the lung.

    The electrocardiogram is helpful in ruling out ischemic heart disease. In patients with pulmonary emboli, the electrocardiogram usually demonstrates sinus tachycardia or is normal. Only in the presence of massive embolization is a right axis deviation and an S1, Q3, T3 pattern seen.

    These test results help narrow the differential diagnosis. The chest radiograph findings rule out a pneumothorax, and a true bacterial pneumonia is less likely in light of the normal sputum findings. The lack of ischemia on the electrocardiogram makes a primary cardiac abnormality unlikely. With the presentation of shortness of breath, a widened alveolar-arterial oxygen gradient, and chest radiograph findings consistent with an infarction, a pulmonary embolism is now the most likely diagnosis.

  • What is the next step in diagnosing an acute pulmonary embolism?

    CT angiography is indicated and if positive shows filling defects in large- and medium-sized pulmonary arteries. Ventilation/perfusion scans are now reserved for patients who cannot tolerate a dye load due to renal insufficiency or have a known iodine allergy. Measurement of the serum D-dimer, a fibrin degradation product that demonstrates a level below 500 g/L, excludes the diagnosis of pulmonary embolism. Pulmonary angiography is rarely indicated.

    P.370


    If the CT angiogram is inconclusive and the suspicion still high Doppler venous studies of the lower extremities, if positive, may substantiate the need for anticoagulation.

  • What is the acute management of pulmonary embolism?

    The goal of therapy is to prevent further embolic episodes, and heparin is the initial drug of choice for accomplishing this. First, a large intravenous loading bolus should be given, followed by continuous-drip infusion, maintained for at least 5 and often 7 to 10 days. Anticoagulation should not be withheld pending the results of further studies unless the patient's risk of bleeding complications is greater than the clinical suspicion of pulmonary emboli. The partial thromboplastin time should be monitored and the heparin dosage adjusted to keep the time between 1.5 to 2.0 times the control.

    Warfarin is started 24 to 48 hours after heparin therapy has been initiated. During the first 3 days of warfarin therapy, the prothrombin time or INR is increased before the onset of true anticoagulation. Therefore, before discontinuing the heparin, the prothrombin time or INR should be therapeutic (1.5 to 2 times normal) for approximately 2 to 3 days. Low-molecular-weight heparins are indicated for prophylaxis in postoperative patients and probably have a role in the management of acute pulmonary embolism and deep venous thrombosis because they do not require monitoring of the anticoagulation effects.

  • How long should anticoagulation therapy be continued?

    Long-term anticoagulation is usually achieved with warfarin, although low-molecular-weight heparins can also be used. Patients with reversible risk factors that are subsequently eliminated should undergo anticoagulation for a total of 3 months. If this is an initial episode of embolism and the patient has no clear risk factors, treatment should probably be maintained for 3 to 6 months. Finally, those patients with recurrent emboli and nonreversible risk factors (e.g., adenocarcinoma, antiphospholipid syndrome, or factor V Leiden deficiency) should be treated for life. When it is uncertain how long to maintain therapy, impedance plethysmography can help in identifying recurrent deep vein thrombosis.

  • What role would thrombolytic therapy have in this patient?

    The role of thrombolytic agents (streptokinase, urokinase, and tissue plasminogen activator) is yet to be elucidated in the management of acute pulmonary embolism. There appear to be no significant differences between the three agents in the treatment of pulmonary emboli, except for their respective costs. Thrombolytic agents do accelerate the resolution of the pulmonary artery clot, but they have not been clearly shown to improve survival as compared with the results observed for conventional heparin therapy. The only adopted use of these agents is for patients with massive embolism and systemic hypotension. When used, thrombolytic therapy must be followed by a standard course of heparin.

  • When should a vena caval filter be placed?

    The purpose of vena caval filters is both to trap emboli and maintain the patency of the inferior vena cava. These filters are largely viewed as an alternative therapy for thromboembolism when anticoagulation is unacceptable. The three most common indications for filter placement are (a) a contraindication to anticoagulation,

    P.371


    (b) failure of proper anticoagulation to prevent the formation of further emboli, and (c) a complication of anticoagulation therapy.

Suggested Readings

Fishman AP, Kelley MA. Pulmonary thromboembolism (including prophylaxis, treatment, sickle cell disease, and multiple pulmonary thrombi). In: Fishman AP, ed. Pulmonary diseases and disorders, 2nd ed. New York: McGraw-Hill, 1987.

Hurewitz AN, Bergofsky EH. Pulmonary embolism. In: Cherniack RM, ed. Current therapy of respiratory disease. Toronto: BC Decker, 1989:259.

Perrier A, Desmaris S, Goehring C, et al. D-dimer testing for suspected pulmonary embolism in outpatients. Am J Respir Crit Care Med 1997;136:492.

PIOPED Investigators. Value of the ventilation/perfusion scan in acute pulmonary embolism: results of the prospective investigation of pulmonary embolism diagnosis (PIOPED). JAMA 1990;263:2753.

Sarcoidosis

  • What are the symptoms and signs of sarcoidosis?

  • What tests are used to establish the diagnosis?

  • What are the therapeutic options?

Discussion

  • What are the symptoms and signs of sarcoidosis?

    Sarcoidosis is a systemic disorder characterized histologically by the presence of noncaseating granulomas. The granulomas can be found in any tissue, such as the lung, skin, myocardium, central nervous system, and kidneys. The symptoms and signs most commonly seen stem from the involvement of the reticuloendothelial system and the lung. Patients may present with one or more of the following: fatigue; a pigmented papulonodular skin rash; splenomegaly; arthritis; and chest radiographic findings indicating bilateral hilar adenopathy or patchy nodular pulmonary infiltrates, or both. Laboratory abnormalities include anemia, leukopenia, hypercalcemia, elevation of the liver enzyme levels in a cholestatic pattern, and a polyclonal gammopathy.

  • What tests are used to establish the diagnosis?

    The most definitive test to establish the diagnosis of sarcoidosis is tissue biopsy. Sites for biopsy include the skin (if a rash exists) or lung. The sensitivity of bronchoscopy with transbronchial biopsy exceeds 90% in obtaining noncaseating granulomas in patients with sarcoidosis who present with hilar adenopathy and pulmonary infiltrates. However, noncaseating granulomas are only suggestive, but not pathognomonic, evidence for the disease. Other diseases that produce granulomas, such as mycobacterial and fungal diseases, must also be considered. These entities can be ruled out by bronchoscopy with biopsy and bronchoalveolar lavage.

    P.372


    The serum level of the angiotensin-converting enzyme (ACE) is elevated in some patients with sarcoidosis, but this is neither a sensitive nor specific enough finding for it to serve as a diagnostic test. This level is elevated in approximately 66% of patients with sarcoidosis, but also occurs in a variety of disorders such as tuberculosis, coccidioidomycosis, hyperthyroidism, and diabetes mellitus. However, the ACE level has been shown to decrease with therapy, and this may, therefore, be a useful objective measure for monitoring the effectiveness of treatment.

  • What are the therapeutic options?

    Sarcoidosis is a very heterogeneous disease, with approximately one third of patients improving without treatment, one third progressing clinically, and one third remaining in relatively stable condition. Unfortunately, there is no reliable way to predict in which group patients will fall. Factors that suggest an unfavorable prognosis include extensive pulmonary parenchymal involvement, restrictive physiology on pulmonary function testing, an elevated ACE level, involvement of at least three organ systems, and black race. Organ involvement that mandates the institution of therapy includes eye, central nervous system, or cardiac involvement, as well as hypercalcemia.

Treatment consists of corticosteroids, usually prednisone or its equivalent initiated at a dosage of 30 to 40 mg per day. There is no evidence that any one steroid preparation is superior to another. Inhaled corticosteroids have not been found to be beneficial in the treatment of sarcoidosis. Between 80% and 90% of patients respond to steroid therapy. When effective, a clinical and radiographic response is usually witnessed within 2 to 4 weeks. This dosage is usually continued for 1 to 2 months, then gradually tapered over the course of the next 1 to 6 months. Many patients can then discontinue taking steroids, but others require ongoing steroid therapy at a dosage of 10 to 15 mg daily or every other day. The response to therapy is confirmed by symptomatic and radiographic improvement, supported by a decreasing ACE level and stable or improving pulmonary function. Although there is symptomatic and radiographic improvement with corticosteroids, there is little evidence that they influence the natural course of the disease.

Case

A 34-year-old black woman is referred to the pulmonary clinic for evaluation of a 2-month history of dry cough, a rash on her forehead and arms, a 5-pound (2.25-kg) weight loss, and an abnormal chest radiographic study. She has an 8-pack-year smoking history and a history of prior intravenous cocaine use, and 6 months ago traveled to Bakersfield, California, for a vacation.

Physical examination reveals a thin woman in no distress. Her temperature is 99 F (37.2 C), pulse is 80 beats per minute, blood pressure is 110/70 mm Hg, and respiratory rate is 20 breaths per minute. A pigmented, papulonodular rash is present on her forehead and upper arms. Funduscopic findings are normal. Bibasilar crackles are heard on chest examination. Abdominal examination reveals an 8-cm liver and palpable spleen tip. There is no cyanosis, clubbing, or edema on examination of her extremities.

P.373


The chest radiographic study reveals bilateral hilar adenopathy and diffuse alveolar and nodular infiltrates. Laboratory findings are as follows: white blood cell count, 4,000/mm3 with 70% polymorphonuclear leukocytes, 10% monocytes, 2% eosinophils, and 17% lymphocytes; hemoglobin, 11 g/dL; hematocrit, 33%; platelet count, 300,000/ L; normal serum electrolyte levels; calcium, 10 mg/dL; albumin, 3.8 g/dL; and total protein, 8.0 g/dL.

  • What is the differential diagnosis in this patient?

  • What tests should be done to establish the diagnosis in this patient?

    Pulmonary function testing reveals the following lung volumes: total lung capacity, 3.32 L (72% of predicted); thoracic gas volume, 1.63 L (64% of predicted); and residual volume, 0.72 L (59% of predicted). Spirometry shows an FVC of 2.72 L (70% of predicted) and FEV1 of 2.12 L (75% of predicted). The diffusing capacity for carbon monoxide (DLCO) is 15.6 (46% of predicted) and the DLCO/alveolar ventilation (VA) is 4.97 (85% of predicted). Arterial blood gas measurements on room air reveal a pH of 7.41, PaCO2 of 32 mm Hg, PaO2 of 68 mm Hg, and oxygen saturation of 94%.

  • How would you interpret the results of the PFTs?

Case Discussion

  • What is the differential diagnosis in this patient?

    As is true of many pulmonary disorders, the radiographic pattern combined with the patient's clinical history and physical examination findings narrows the differential diagnosis. In a patient who presents with this clinical scenario and normal cellular immunity, the differential diagnosis is broad and includes various indolent infectious processes such as tuberculosis; fungal infections such as histoplasmosis, coccidioidomycosis, and Cryptococcus neoformans infection; idiopathic immunologic disorders such as sarcoidosis; and, less commonly, metastatic neoplastic disease, Hodgkin's disease, non Hodgkin's lymphoma, and occupational lung diseases such as berylliosis and silicosis. However, in a patient infected with HIV, the differential diagnosis includes a higher probability of mycobacterial infection, fungal infection, and Kaposi's sarcoma.

  • What tests should be done to establish the diagnosis in this patient?

    A serum HIV test should be done in this patient because of her history of intravenous drug abuse. For most people with possible sarcoidosis, however, this test is not necessary.

    A sputum sample should be obtained for acid-fast staining and mycobacterial culture. In patients with extensive pulmonary infiltrates due to M. tuberculosis infection, three separate morning sputum samples are highly sensitive for detecting the pathogen. In most healthy hosts with active pulmonary tuberculosis, the PPD (purified protein derivative) skin test result is positive. Patients with sarcoidosis are frequently anergic in response to a variety of skin tests, including the PPD test.

    Sputum specimens for fungal staining and culture should also be obtained. The fungi that mimic sarcoidosis are restricted to certain endemic areas. For instance,

    P.374


    histoplasmosis is found in the midwestern and southeastern United States. Coccidioidomycosis is endemic to the desert southwest and arid regions of California, such as the Mojave Desert and the San Joaquin Valley. Given the patient's recent trip to Bakersfield, California, it is necessary to exclude possible infection with coccidioidomycosis. A thorough travel and occupational history should always be taken to exclude any atypical fungal exposure.

    Tissue should also be obtained for the purpose of excluding infection and neoplasm, and to support the diagnosis of sarcoidosis. As already discussed, skin biopsy or bronchoscopy with transbronchial biopsy would be helpful if the results revealed noncaseating granulomas. Acid-fast and silver staining can be performed on the biopsy specimens to exclude mycobacterial and fungal infections. For this patient, most clinicians would recommend fiberoptic bronchoscopy with bronchoalveolar lavage and transbronchial biopsy. In the setting of sarcoidosis, the bronchoalveolar lavage fluid characteristically shows an increased percentage of lymphocytes with a predominant CD4 phenotype. In addition, stains and cultures for mycobacterial and fungal diseases can be performed on the bronchoalveolar lavage fluid.

  • How would you interpret the results of the PFTs?

    The PFTs show restricted lung volumes. Spirometry shows a mild degree of obstruction, particularly given the underlying restrictive physiology. The DLCO is reduced.

    PFTs, including lung volumes, DLCO, spirometry, and arterial blood gas measurement, should be performed for every patient with sarcoidosis and pulmonary parenchymal involvement. The most frequent abnormalities encountered are a reduction in lung volumes (restrictive physiology), often accompanied by a reduction in DLCO. It is also not uncommon to find reduced expiratory flow rates, indicating airway involvement with sarcoidosis. The arterial oxygen saturation usually remains relatively normal at rest, unless advanced disease is present. With exercise, the PaO2 frequently falls.

Suggested Readings

Gilman MJ, Wang KP. Transbronchial lung biopsy in sarcoidosis. Am Rev Respir Dis 1980;122:721.

Hillerdal G, Nou E, Osterman K, et al. Sarcoidosis: epidemiology and prognosis: a 15-year European study. Am Rev Respir Dis 1984;130:29.

Rust M, Bergmann L, Kuhn T. Prognostic value of chest radiograph, serum angiotensin-converting enzyme and T helper cell count in blood and in bronchoalveolar lavage of patients with pulmonary sarcoidosis. Respiration 1985;48:231.

Tuberculosis

  • What is the contemporary epidemiology of tuberculosis?

  • What symptoms and radiographic features are associated with tuberculosis?

  • Who should receive treatment (prophylaxis) for tuberculosis infection?

P.375


Discussion

  • What is the contemporary epidemiology of tuberculosis?

    Despite numerous medical advances in the past century, tuberculosis is still the cause of at least 1 million deaths worldwide each year, and its incidence, which was increasing in the 1980s and early 1990s primarily because of AIDS, is again decreasing. Elderly patients now constitute nearly half of the newly diagnosed cases of tuberculosis in the United States because these people were exposed to the tuberculosis epidemic in the first quarter of the 20th century and have been harboring latent infection for many decades. The case fatality rate in the elderly is also disproportionately high, and they face a higher risk of complications with treatment. Others at risk for tuberculosis include medically underserved, low-income, ethnic minority populations, especially African Americans, Native Americans, and Hispanics; institutionalized people; patients with chronic renal failure, silicosis, diabetes mellitus, or lymphoreticular malignancies; alcoholics or those with other substance abuse habits; those with malnutrition; those who have undergone gastrectomy; and those undergoing immunosuppressive or long-term corticosteroid therapy.

  • What symptoms and radiographic features are associated with tuberculosis?

    Diversity characterizes the clinical manifestations of tuberculosis. Although many patients have constitutional symptoms consisting of weight loss, fatigue, fever, and night sweats, as well as pulmonary symptoms such as cough, intermittent hemoptysis, chest pain, and dyspnea, none of these is uniformly present. In addition, the elderly and patients with AIDS often have extrapulmonary disease and the symptoms and signs are atypical.

    The classic chest radiograph in an adult with pulmonary tuberculosis demonstrates fibronodular infiltration of the posterior or apical segments of the upper lobe. There may also be cavitation. Tuberculosis can, however, produce almost any form of pulmonary radiographic abnormality. Moreover, normal radiographic findings do not exclude a diagnosis of disseminated tuberculosis in an elderly or immunocompromised patient. Hilar adenopathy on a chest radiograph in a patient seropositive for HIV is considered tuberculosis until proved otherwise.

  • Who should receive treatment (prophylaxis) for tuberculosis infection?

    The tuberculin skin test is the traditional method of demonstrating infection with M. tuberculosis, and is based on the principle that infection elicits delayed-type hypersensitivity to certain antigens in culture extracts called tuberculins. The tuberculin most commonly used is PPD; it is injected intracutaneously on the volar aspect of the forearm in a dose of 5 tuberculin units. Induration of the site at 48 to 72 hours indicates delayed hypersensitivity to infection with M. tuberculosis, but does not necessarily signify the presence of active disease, only infection.

Preventive therapy with isoniazid (INH) given for 6 to 12 months clearly decreases the risk of future tuberculosis in other words, the progression from an infected state to an actively diseased state manifesting the clinical, radiographic, and

P.376


microbiologic profile. The goal of INH monotherapy is therefore to treat subclinical infection brought to light by the positive result of tuberculin skin test. By strict definition, it is not true prophylaxis, but it is often referred to as such. The dosage of INH in adults is 5 mg/kg, up to a total of 300 mg orally per day. People who have contact with a patient having newly diagnosed pulmonary tuberculosis, and whose tuberculin skin test result is positive, should undergo INH preventive therapy. If a chest radiograph shows inactive parenchymal tuberculosis (upper lobe scarring), the skin test result is positive, and active disease has been excluded by negative sputum findings, such patients should receive INH therapy. People younger than 35 years with a positive result on skin test and a normal chest radiograph should also be treated. Patients whose skin test result is positive and in whom the following clinical situations apply should receive 6 to 12 months of preventive therapy: HIV positivity; silicosis; diabetes mellitus, especially poorly controlled insulin-dependent diabetes; steroid therapy, especially more than 15 mg of prednisone per day; chronic renal failure; lymphoreticular malignancies: leukemia, lymphoma, and Hodgkin's disease; immunosuppressive therapy; gastrectomy; jejunoileal bypass; and weight loss of 10% or more of the ideal body weight.

Case

A 68-year-old African-American man with a long history of tobacco and ethanol abuse is brought in by his family for evaluation of weight loss, low-grade fevers, and failure to thrive. The patient reports a 2- to 3-month history of progressive 20-pound (9-kg) weight loss, as well as fevers, nonproductive cough, and generalized weakness. His cough became productive of white sputum 2 days earlier. The patient denies chest pain, hemoptysis, ill contacts, recent travel, or HIV risk factors. He has smoked one pack of cigarettes per day for 40 years. He drinks approximately one pint (half a liter) of alcohol a day and has done so for many years. Careful review of his old records reveals that a PPD test was positive approximately 12 years ago.

Physical examination discloses a cachectic, ill-appearing, elderly man in moderate respiratory distress. His oral temperature is 38.5 C (101.3 F), with a respiratory rate of 30 breaths per minute, heart rate of 126 beats per minute, and blood pressure of 100/60 mm Hg. Physical examination findings are remarkable for poor dentition, and rales and rhonchi throughout the right chest, but otherwise negative. Initial laboratory evaluation reveals a white blood cell count of 16,000/mm3 with a leftward shift and a hematocrit of 35%. His oxygen saturation is 80% on room air. A chest radiograph shows a large interstitial and alveolar infiltrate with air bronchograms in the right upper lobe; no cavitation, pleural effusion, or volume loss is noted. Arterial blood gas measurements on the night of admission show worsening hypoxemia and hypercapnia.

The patient is placed in respiratory isolation and is electively intubated on the night of admission. Examination of his sputum, done the next morning, reveals a pathogen.

  • What is the differential diagnosis of this patient's respiratory failure?

  • What is the diagnostic strategy at this point in his illness?

  • What is the correct therapeutic plan?

P.377


Case Discussion

  • What is the differential diagnosis of this patient's respiratory failure?

    This patient's history is consistent with an infectious process, perhaps superimposed on an underlying malignancy. Bacterial pathogens such as S. pneumoniae, Legionella pneumophila, and H. influenzae are possible causes. Because of his history of ethanol abuse and poor dentition, aspiration pneumonia (with anaerobic pathogens) is also a possibility, but the location of the infiltrates in the upper lobe makes this less likely as aspiration pneumonia tends to favor dependent portions of the lung, such as the superior segments of both lower lobes. Bacterial pneumonia distal to an obstructing endothelial lesion is clearly a possibility given his long-standing tobacco use and age; however, the absence of volume loss and presence of air bronchograms on his radiograph somewhat militate against this diagnosis. Viral infection is less likely, given his history and the lobar infiltrate. Fungal infection, particularly with Histoplasma capsulatum, is a possibility, but less likely given his negative geographic history and the absence of adenopathy on the radiograph. He has no HIV risk factors.

    An important clue to the diagnosis is the positive result of PPD skin test. Because most active cases of pulmonary tuberculosis in adults constitute either postprimary disease or the reactivation of a protracted, even lifelong, infection with the tubercle bacillus, this plus his positive PPD test result point to the diagnosis. The important point here is that the clinician should suspect reactivation of tuberculosis even without a history of a positive PPD test result. This patient has several currently recognized epidemiologic risk factors for tuberculosis: he is elderly, African American, and a substance abuser (ethanol), and he has poor nutritional status. He has an upper lobe infiltrate. The absence of an upper lob infiltrate, however, does not rule out tuberculosis, because the classic fibronodular infiltration of the posterior or apical segments of the upper lobe (right greater than left) may not be present in the elderly or, especially, in HIV-infected patients. Tuberculosis probably needs to be considered in the differential diagnosis of pneumonia in every patient older than 60 years. This patient's respiratory failure with hypoxemia and respiratory muscle fatigue (hypercapnia) could be due to any one of the processes discussed, but tuberculosis is the most likely diagnosis.

  • What is the diagnostic strategy at this point in his illness?

    A careful search for the pathogen, or pathogens, should be undertaken. Blood cultures, Gram's staining and culture of sputum samples, plus sputum stains for acid-fast organisms (mycobacteria) and the possibility of Legionella indicate that a fluorescent antibody test should be performed. The value of sputum examination cannot be overemphasized. Gram's staining, especially of a tracheal aspirate or sputum produced by a strong deep cough, can help in the diagnosis of virtually any bacterial infectious agent in the differential diagnosis. (Legionella is difficult to see, but a large number of neutrophils without organisms suggest this diagnosis.) In addition to the stains for acid-fast bacilli (AFB), tuberculosis can be diagnosed with a fluorochrome technique called auramine O. This patient's sputum was positive for AFB. Approximately 104 organisms/mL of sputum are required for an AFB smear to be

    P.378


    positive, and only 50% to 80% of patients with pulmonary tuberculosis have positive sputum smear findings. A rapid radiometric technique known as BACTEC allows the recovery and identification of tuberculosis in 10 days, which is an advantage over conventional culture methods that can take 3 to 6 weeks to grow mycobacteria.

    A PPD test with two controls should be placed and an induration of 10 mm or more is considered a positive result. However, in the setting of HIV infection HIV risk factors, recent close contact with infectious people, or chest radiographic findings consistent with old healed tuberculosis (upper lobe scarring) an induration of 5 mm or more is considered a positive reaction. However, as already mentioned, a negative result does not exclude tuberculosis because up to 25% to 30% of newly diagnosed patients with tuberculosis have a negative ( 9 mm) skin test result. A booster effect is more powerful ( 6 mm increase) in the cutaneous reaction, and is achieved by performing a second PPD test 7 to 10 days after the first. A second PPD should therefore be considered in this patient, but especially in an elderly patient with a negative PPD, in whom the clinical suspicion for tuberculosis is high. However, the sine qua non test for tuberculosis remains the sputum smear and culture.

    Additional diagnostic tests that should be part of the evaluation of this patient include an HIV test and comparison of previous and current chest radiographs, if possible. The latter would constitute an important part of the evaluation in this patient. Fiberoptic bronchoscopy could also help in determining if there is an endobronchial lesion, or extrinsic compression from a mass, as well as enable sampling of secretions and biopsies for microbiologic evaluation.

  • What is the correct therapeutic plan?

    After appropriate culture results are obtained, treatment needs to be directed toward the most likely pathogens in this man's illness. On the basis of the differential diagnosis, you would need to provide antibiotic coverage for S. pneumoniae, H. influenzae, possibly anaerobes, as well as Legionella species. The severity of this patient's illness mandates aggressive, but well-considered, therapy. His initial regimen should include coverage for community-acquired pneumonia and Legionella. Antituberculous therapy should be initiated only after his AFB smears prove positive. It was also very important that on admission the patient was placed in respiratory isolation to prevent the spread of tuberculosis, which is transmitted almost exclusively by means of aerosolized respiratory secretions, not only to other patients but to health care workers as well. His sputum smear positivity for M. tuberculosis constitutes a state of infectiousness. People receiving therapy promptly become noninfectious as their cough subsides and the concentration of organisms in their sputum decreases. Most authorities believe that treatment reverses infectiousness within approximately 2 weeks of the start of therapy; until then, isolation measures should be maintained.

    His antituberculous chemotherapy should consist of a four-drug regimen, comprising INH, rifampin, pyrazinamide, and streptomycin, and be maintained for 6 months. Many studies have convincingly demonstrated the curative efficacy of multidrug regimens given for 6 to 9 months. A standard treatment is effective when given for 6 months in a supervised setting. Compliance with antituberculous therapy is absolutely critical for cure. Noncompliance is also one of the major reasons for the emergence of drug resistance. Multidrug therapy is always used in the treatment of

    P.379


    tuberculosis because of the potential for primary or spontaneous resistance, which occurs in 1 106 organisms. Extrapulmonary tuberculosis is treated like pulmonary tuberculosis, and tuberculosis contracted in the setting of AIDS is also treated with standard regimens, and is still curable.

Suggested Readings

American Thoracic Society/Centers for Disease Control and Prevention/Infectious Disease Society of America: Controlling tuberculosis in the United States. Am J Respir Crit Care Med 2005;172:1169 1227; Treatment of tuberculosis. Am J Respir Crit Care med 2003;167:603 662.

Bass JB, Farer LS, Hopewell PC, et al. American Thoracic Society, Medical Section of the American Lung Association. Treatment of tuberculosis and tuberculous infection in adults and children. Am Rev Respir Dis 1986;134:355.

Bass JB, Farer LS, Hopewell PC, et al. American Thoracic Society, Medical Section of the American Lung Association. Diagnostic standards and classification of tuberculosis. Am Rev Respir Dis 1990;142:725.

Cohn DL, Catlin BJ, Peterson KL, et al. A 62-dose, 6-month therapy for pulmonary and extrapulmonary tuberculosis: a twice-weekly, directly observed, and cost-effective regimen. Ann Intern Med 1990;112:407.

Heffner JE, Strange C, Sahn SA. The impact of respiratory failure on the diagnosis of tuberculosis. Arch Intern Med 1988;148:1103.

Iseman M. Tuberculosis. In: Synopsis of clinical pulmonary disease, 4th ed. St. Louis: Mosby Year Book, 1989.



Internal Medicine Casebook. Real Patients, Real Answers
The Internal Medicine Casebook: Real Patients, Real Answers
ISBN: 0781765293
EAN: 2147483647
Year: 2007
Pages: 14

flylib.com © 2008-2017.
If you may any questions please contact us: flylib@qtcs.net